Mental Health

Case Study, Mohr

CHAPTER 1, Introduction to Psychiatric–Mental Health Nursing In completing the case study, students will be addressing the following learning objectives: Explain elements that contribute to mental health.

Review trends, problems, and goals related to the delivery of mental health care and treatment of mental illness.

1. Karen is a 25-year-old white woman who lives alone in an apartment with her dog. Karen has been divorced for 2 years and is taking Prozac prescribed by her psychiatrist for depression.  Karen and her boyfriend had been discussing marriage until he told her that he wanted to end their relationship.  Karen became even more depressed and could not work for a week.  Karen returned to work, refusing to discuss her issues with family, friends, or coworkers. She did, however, make an appointment to see a psychiatric nurse practitioner.  Karen told the nurse that she was making some changes in her life.  Karen said that she and a girlfriend were joining a gym program for workouts and a social group for young men and women.  Karen stated that she realizes that her former boyfriend had not been committed to her, and she anticipates meeting and dating other young men from the adult social group.  Karen also said that she thinks that the gym exercise will be beneficial to her mentally and physically. Mental Health

Learning Objectives: 1, 4)

  • How will the psychiatric nurse assess if Karen has made progress toward self-realization? 
  •  
  • Identify strengths that Karen has for progress in personal growth.
  •  

Karen has been seeking treatment for her depression. Analyze the factors that might contribute to Karen’s reluctance to discuss her depression with others.

Case Study, Mohr:  CHAPTER 2, Neuroscience: Biology and Behavior

In completing the case study, students will be addressing the following learning objectives:

Discuss neuroplasticity and how this concept relates to mental health and mental illness.

Briefly explain the importance of interaction between genes and environment, the role of endophenotypes, and the stress-diathesis model of psychiatric illness.

1. Michael is a 22-year-old college senior whose GPA has declined with this semester’s grades.  Michael plans to apply to medical school and thinks that the lower GPA may prevent his acceptance to medical school.  For the last 2 weeks, Michael has skipped most classes because he has insomnia and fatigue.  Michael is now very depressed and has been thinking of suicide.  He took a loaded gun from his father’s gun cabinet and then wrote a suicide note to his family.  At the last moment, he telephoned 911 and told them of his suicide plan.  The police came, took the gun away, and then took Michael to the city hospital to be admitted for psychiatric treatment.   In the admission interview with the psychiatric nurse, Michael said that his pastor thought that only weak-willed people experienced depression and that it was a punishment for personal sins and the sins of one’s ancestors.  Michael told the nurse that he must be weak-willed and will never be able to accomplish anything.  The psychiatric nurse explained that multiple factors are the cause of depression.  The nurse told Michael that one theory holds that an imbalance of neurotransmitters, or chemical messengers of the brain, occurs in depression.  Neurotransmitters influence the individual’s emotions, thoughts, and subsequent behavior.  Recent research implies that neurobiology, heredity, as well as Psychological and environmental factors may be involved in the development and progression of depression Mental Health. 

(Learning Objectives: 5, 6)

a. Will Michael think that the psychiatric nurse’s explanation for the cause of depression is more correct than that of his pastor?

b. Michael asks the nurse why he has to have psychotherapy. He states that he only needs to take a couple of pills to get better. How should the nurse respond to Michael’s question and comment?

c. Develop an assessment question for each of the following possible causes of Michael’s    development of depression:

              Genetic, Environment and Stress.

Case Study, Mohr

CHAPTER 3, Conceptual Frameworks and Theories:

In completing the case study, students will be addressing the following learning objective:

Give examples of behavioral and cognitive-behavioral interventions.

The student nurse has been assigned a 37-year-old woman admitted to the psychiatric hospital with an anxiety disorder.  This morning, the student notices that the client has a tense facial expression and is walking constantly around the group room.   The student walked over to the client and used reflective communication by stating, “I see that you have a tense expression and are walking around almost all of the time.  Is there something that we could discuss?”  The client replied that she has talked on the telephone to her mother who was keeping her children while she was in the hospital.  The client said that her mother had told her that she was not a good mother, and then said, “I guess I am a bad mother, but I could never measure up to my mother’s expectations.”  The student has learned that negative self-talk can greatly aggravate anxiety and lead to depression.  The student decided to use a behavioral intervention with the client and asked the client who is a good artist and why the client liked the artist’s works. The student and the client then made a list of activities that the client liked. The student taught the client to engage in one of these activities when an unpleasant experience evoked negative thoughts.  The following day, the student decided that the client needed some cognitive restructuring for her relationship with her mother.  The student taught the client that during discussions with her mother, feelings of incompetence might be experienced when the mother made negative comments.  The client was instructed that if her mother made negative comments about parenting, she was to immediately tell her mother that she was a good parent to her children and terminate the conversation at the first opportunity. Mental Health

ORDER A FREE-PLAGIARISM PAPER HERE

(Learning Objective: 3)

a. Will the client be able to learn cognitive restructuring in her relationship with her mother?

  b.    How will engaging in activities help the client with her anxiety?

Case Study, Mohr

CHAPTER 4, Evidence-Based Practice

 In completing the case study, students will be addressing the following learning objective:

Define evidence-based care.

1. Jessica, a 17-year-old girl who came with her family from Romania to the United States 10 years ago, is brought to the mental health clinic by her mother.  At the admission psychiatric interview, the mother stated that Jessica has been saying for 6 months that aliens have been conducting experiments on her and will soon take her in a spaceship to their planet.  Jessica is often awake at night and roams the house with a hammer and sharp knife, searching for aliens. Jessica’s mother says that she fears that some night Jessica will harm a family member whom she believes to be an alien.  The mother then said that she did not bring Jessica for psychiatric care earlier because she had heard that much of mental health treatment was quackery and a waste of money. The psychiatric nurse explains to Jessica’s mother that all mental health treatment is based on scientific principles. 

(Learning Objective: 3)

a. How would the nurse best explain that the care Jessica will receive at the psychiatric   facility is based on evidence? Give two examples of psychiatric evidence-based care, and explain the scientific evidence that supports your examples.

Case Study, Mohr

CHAPTER 5, Legal and Ethical Aspects: In completing the case study, students will be addressing the following learning objectives:  Identify the basic rights of people with mental illness. Describe the different types of commitments and states of competency.

1. Gerald, a 22-year-old black man, has a diagnosis of schizoaffective disorder.  Gerald has been living at home, but this afternoon he had a physical fight with the neighbors and set a fire in their garage to burn their house.  His father took him to the local psychiatric hospital and said that he wants an involuntary commitment for Gerald since he will not agree to the hospitalization.  The father expressed concern over Gerald’s legal rights.  The psychiatric nurse practitioner recommends to the father that he commit Gerald on involuntarily for emergency care for a period of 72 hours since he is clearly a danger to others. The nurse explains that Gerald will be evaluated to determine if he needs involuntary detention for observation and treatment for a longer period of time.  The nurse explains that Gerald will be periodically evaluated, and that when it is determined that he is no longer a danger to others or himself, Gerald will be released from the hospital. Mental Health

(Learning Objectives: 4, 5)

a. What basic legal rights does Gerald have at the time of this illness?

b. Why can Gerald not make the decision for himself about being an involuntary emergency commitment to the hospital?

Case Study, Mohr

CHAPTER 6, Culture

In completing the case study, students will be addressing the following learning objective:

Describe skills essential to the implementation of culturally competent care.

1. Mrs. Maria Gonzalez is a Mexican National, age 58, who was brought to a health clinic by her adult son. She is complaining that she has had aches and pains all over her body for 2 days. Mrs. Gonzalez states that her neighbor is a witch who gave her the mal do ojo (evil eye) and cast a spell on her to cause her death. The clinic nurse takes an oral temperature and it is 101º F.  The clinic nurse refers Mrs. Gonzales to the clinic physician because she believes that the client has influenza.  Mrs. Gonzales is reluctant to see the doctor and states that the doctor cannot prevent her death. 

(Learning Objective: 8)

a. How can the nurse provide culturally competent care to Mrs. Gonzalez and convince her to see the clinic doctor? 

b. Can Mrs. Gonzalez be convinced that there is no spell cast on her to cause her death? Explain your answer.

Case Study, Mohr

CHAPTER 7, Spirituality in Psychiatric Care

In completing the case study, students will be addressing the following learning objective: Explore reasons for the resurgence of interest in the spiritual aspects of healthcare.

  • Marta, a 16-year-old high school teenager, has been depressed over a recent break-up with her boyfriend.  Marta, accompanied by her mother, is seen in the community mental health clinic.  A mild antidepressant is prescribed for Marta with weekly counseling sessions.  Marta and her mother ask if it would be beneficial for Marta to attend a 1-week church camp to begin in 2 months.  Marta would like to reconnect with her religion and voices a belief that the camp experience will help her. The psychiatric nurse practitioner informs them that research reveals that involvement in religious activities has been helpful in social support and in coping with depression.  Marta appears relieved and said that she will register for the church camp.

(Learning Objective: 2)

a. Why might Marta want to reconnect with her religion, and how will attendance at the church camp benefit Marta?

b. If Marta attends the church camp, will this pose a problem to counseling sessions?

Case Study, Mohr

CHAPTER 8,- Nursing Values, Attitudes, and Self-Awareness

In completing the case study, students will be addressing the following learning objective:

Discuss how situational factors can potentially influence the behavior of healthcare professionals toward clients with mental illness.

1. Joe, a 26-year-old Caucasian man, is a client in a state prison system.  Joe is admitted to the prison clinic after being involved in a fight in which he sustained a stab wound to the chest that did not penetrate the lungs or major blood vessels.  The clinic doctor on duty was an employee of several years at the prison.  The doctor showed little compassion for Joe, stating, “He is a convicted criminal, and he is just getting back some of what he deserves.”  The new graduate nurse who was being oriented to the clinic thought that the doctor did not exhibit professional behavior toward Joe.  The clinic nursing supervisor later explained that the doctor was influenced by situational factors. Mental Health

(Learning Objective: 3)

  • What is a situational factor, and how can this influence the behavior of healthcare workers? Provide an example where a situational factor impacted on your behavior while in a healthcare setting.

Case Study, Mohr

CHAPTER 9, the Nursing Process in Psychiatric–Mental Health Care

In completing the case study, students will be addressing the following learning objective:

Apply the nursing process to psychiatric–mental health nursing.

1. The student nurse is assigned to assist the psychiatric nurse with the admission interview of a client at the psychiatric hospital.  The nurse explains to the student that the interview is very important in obtaining a total health history of the client.  The nurse should be courteous and respectful of the client to obtain as much information from the client as possible.  Assessment information should include the subjective information from the client with the reason for needing treatment, the cause of the present problem(s), and the client’s expectation of the outcome of treatment regimen.  The nurse alerts the student to the need to be sensitive to both verbal and nonverbal behavior of the client and to focus on topics that seem important to the client.

(Learning Objective: 1)

a. How can attention to the client’s nonverbal cues be of value in an interview?

b. Why is the interview the most important aspect of the nursing process?

Case Study, Mohr

CHAPTER 10, the Interview and Assessment Process

In completing the case study, students will be addressing the following learning objective:

Describe essential elements to include in psychiatric interviews and the assessment process.

  • The student nurse is accompanying the psychiatric nurse during the nursing interview and assessment of a newly admitted patient.  The psychiatric nurse told the student that preparation with subjective and objective data collection is an important part of the process.  The nurse explains that assessment has reference to the interviewer’s interpretation and prioritization of all data for the client.  The nurse must have self-awareness and self-knowledge to be objective and avoid influencing the responses of the client.  Anxiety on the part of the nurse may limit the ability for thorough data collection and interpretation.  Anxiety in the nurse may evoke anxiety on the part of the client. The psychiatric nurse stressed that a process recording, or written analysis of the interaction between the client and nurse, is essential for nurses to recognize the effects of their communication style in the assessment process.  A review of the client’s history is important, and a private setting for the interview is necessary.  The content of the nursing assessment should include the ability and reliability of the client’s response to questions of the interviewer and the skill of the nurse in identification of relevant facts.  The nurse should discuss with the client prior health history, any present illness, and the reason for seeking healthcare at this time.  Medication history with compliance and allergies of the client require investigation.   Substance use by the client, past illnesses, and family history need exploration.

(Learning Objective: 2)

a   The student nurse asked the clinical psychiatric nurse practitioner to explain what she considered the most essential part of the assessment process. How should the psychiatric nurse respond to the student’s question? Explain your answer. Mental Health

Mental Health

Case Study, Mohr

CHAPTER 1, Introduction to Psychiatric–Mental Health Nursing In completing the case study, students will be addressing the following learning objectives: Explain elements that contribute to mental health.

Review trends, problems, and goals related to the delivery of mental health care and treatment of mental illness.

1. Karen is a 25-year-old white woman who lives alone in an apartment with her dog. Karen has been divorced for 2 years and is taking Prozac prescribed by her psychiatrist for depression.  Karen and her boyfriend had been discussing marriage until he told her that he wanted to end their relationship.  Karen became even more depressed and could not work for a week.  Karen returned to work, refusing to discuss her issues with family, friends, or coworkers. She did, however, make an appointment to see a psychiatric nurse practitioner.  Karen told the nurse that she was making some changes in her life.  Karen said that she and a girlfriend were joining a gym program for workouts and a social group for young men and women.  Karen stated that she realizes that her former boyfriend had not been committed to her, and she anticipates meeting and dating other young men from the adult social group.  Karen also said that she thinks that the gym exercise will be beneficial to her mentally and physically.

Learning Objectives: 1, 4)

  • How will the psychiatric nurse assess if Karen has made progress toward self-realization? 
  •  
  • Identify strengths that Karen has for progress in personal growth.
  •  

Karen has been seeking treatment for her depression. Analyze the factors that might contribute to Karen’s reluctance to discuss her depression with others.

Case Study, Mohr:  CHAPTER 2, Neuroscience: Biology and Behavior

In completing the case study, students will be addressing the following learning objectives:

Discuss neuroplasticity and how this concept relates to mental health and mental illness.

Briefly explain the importance of interaction between genes and environment, the role of endophenotypes, and the stress-diathesis model of psychiatric illness.

1. Michael is a 22-year-old college senior whose GPA has declined with this semester’s grades.  Michael plans to apply to medical school and thinks that the lower GPA may prevent his acceptance to medical school.  For the last 2 weeks, Michael has skipped most classes because he has insomnia and fatigue.  Michael is now very depressed and has been thinking of suicide.  He took a loaded gun from his father’s gun cabinet and then wrote a suicide note to his family.  At the last moment, he telephoned 911 and told them of his suicide plan.  The police came, took the gun away, and then took Michael to the city hospital to be admitted for psychiatric treatment.   In the admission interview with the psychiatric nurse, Michael said that his pastor thought that only weak-willed people experienced depression and that it was a punishment for personal sins and the sins of one’s ancestors.  Michael told the nurse that he must be weak-willed and will never be able to accomplish anything.  The psychiatric nurse explained that multiple factors are the cause of depression.  The nurse told Michael that one theory holds that an imbalance of neurotransmitters, or chemical messengers of the brain, occurs in depression.  Neurotransmitters influence the individual’s emotions, thoughts, and subsequent behavior.  Recent research implies that neurobiology, heredity, as well as Psychological and environmental factors may be involved in the development and progression of depression. 

(Learning Objectives: 5, 6)

a. Will Michael think that the psychiatric nurse’s explanation for the cause of depression is more correct than that of his pastor?

b. Michael asks the nurse why he has to have psychotherapy. He states that he only needs to take a couple of pills to get better. How should the nurse respond to Michael’s question and comment?

c. Develop an assessment question for each of the following possible causes of Michael’s    development of depression:

              Genetic, Environment and Stress.

Case Study, Mohr

CHAPTER 3, Conceptual Frameworks and Theories:

In completing the case study, students will be addressing the following learning objective:

Give examples of behavioral and cognitive-behavioral interventions.

The student nurse has been assigned a 37-year-old woman admitted to the psychiatric hospital with an anxiety disorder.  This morning, the student notices that the client has a tense facial expression and is walking constantly around the group room.   The student walked over to the client and used reflective communication by stating, “I see that you have a tense expression and are walking around almost all of the time.  Is there something that we could discuss?”  The client replied that she has talked on the telephone to her mother who was keeping her children while she was in the hospital.  The client said that her mother had told her that she was not a good mother, and then said, “I guess I am a bad mother, but I could never measure up to my mother’s expectations.”  The student has learned that negative self-talk can greatly aggravate anxiety and lead to depression.  The student decided to use a behavioral intervention with the client and asked the client who is a good artist and why the client liked the artist’s works. The student and the client then made a list of activities that the client liked. The student taught the client to engage in one of these activities when an unpleasant experience evoked negative thoughts.  The following day, the student decided that the client needed some cognitive restructuring for her relationship with her mother.  The student taught the client that during discussions with her mother, feelings of incompetence might be experienced when the mother made negative comments.  The client was instructed that if her mother made negative comments about parenting, she was to immediately tell her mother that she was a good parent to her children and terminate the conversation at the first opportunity.

(Learning Objective: 3)

a. Will the client be able to learn cognitive restructuring in her relationship with her mother?

  b.    How will engaging in activities help the client with her anxiety? Mental Health

Case Study, Mohr

CHAPTER 4, Evidence-Based Practice

 In completing the case study, students will be addressing the following learning objective:

Define evidence-based care.

1. Jessica, a 17-year-old girl who came with her family from Romania to the United States 10 years ago, is brought to the mental health clinic by her mother.  At the admission psychiatric interview, the mother stated that Jessica has been saying for 6 months that aliens have been conducting experiments on her and will soon take her in a spaceship to their planet.  Jessica is often awake at night and roams the house with a hammer and sharp knife, searching for aliens. Jessica’s mother says that she fears that some night Jessica will harm a family member whom she believes to be an alien.  The mother then said that she did not bring Jessica for psychiatric care earlier because she had heard that much of mental health treatment was quackery and a waste of money. The psychiatric nurse explains to Jessica’s mother that all mental health treatment is based on scientific principles.

ORDER A FREE-PLAGIARISM PAPER HERE

(Learning Objective: 3)

a. How would the nurse best explain that the care Jessica will receive at the psychiatric   facility is based on evidence? Give two examples of psychiatric evidence-based care, and explain the scientific evidence that supports your examples.

Case Study, Mohr

CHAPTER 5, Legal and Ethical Aspects: In completing the case study, students will be addressing the following learning objectives:  Identify the basic rights of people with mental illness. Describe the different types of commitments and states of competency.

1. Gerald, a 22-year-old black man, has a diagnosis of schizoaffective disorder.  Gerald has been living at home, but this afternoon he had a physical fight with the neighbors and set a fire in their garage to burn their house.  His father took him to the local psychiatric hospital and said that he wants an involuntary commitment for Gerald since he will not agree to the hospitalization.  The father expressed concern over Gerald’s legal rights.  The psychiatric nurse practitioner recommends to the father that he commit Gerald on involuntarily for emergency care for a period of 72 hours since he is clearly a danger to others. The nurse explains that Gerald will be evaluated to determine if he needs involuntary detention for observation and treatment for a longer period of time.  The nurse explains that Gerald will be periodically evaluated, and that when it is determined that he is no longer a danger to others or himself, Gerald will be released from the hospital.

(Learning Objectives: 4, 5)

a. What basic legal rights does Gerald have at the time of this illness?

b. Why can Gerald not make the decision for himself about being an involuntary emergency commitment to the hospital?

Case Study, Mohr

CHAPTER 6, Culture

In completing the case study, students will be addressing the following learning objective:

Describe skills essential to the implementation of culturally competent care.

1. Mrs. Maria Gonzalez is a Mexican National, age 58, who was brought to a health clinic by her adult son. She is complaining that she has had aches and pains all over her body for 2 days. Mrs. Gonzalez states that her neighbor is a witch who gave her the mal do ojo (evil eye) and cast a spell on her to cause her death. The clinic nurse takes an oral temperature and it is 101º F.  The clinic nurse refers Mrs. Gonzales to the clinic physician because she believes that the client has influenza.  Mrs. Gonzales is reluctant to see the doctor and states that the doctor cannot prevent her death. 

(Learning Objective: 8)

a. How can the nurse provide culturally competent care to Mrs. Gonzalez and convince her to see the clinic doctor? 

b. Can Mrs. Gonzalez be convinced that there is no spell cast on her to cause her death? Explain your answer.

Case Study, Mohr

CHAPTER 7, Spirituality in Psychiatric Care

In completing the case study, students will be addressing the following learning objective: Explore reasons for the resurgence of interest in the spiritual aspects of healthcare.

  • Marta, a 16-year-old high school teenager, has been depressed over a recent break-up with her boyfriend.  Marta, accompanied by her mother, is seen in the community mental health clinic.  A mild antidepressant is prescribed for Marta with weekly counseling sessions.  Marta and her mother ask if it would be beneficial for Marta to attend a 1-week church camp to begin in 2 months.  Marta would like to reconnect with her religion and voices a belief that the camp experience will help her. The psychiatric nurse practitioner informs them that research reveals that involvement in religious activities has been helpful in social support and in coping with depression.  Marta appears relieved and said that she will register for the church camp.

(Learning Objective: 2)

a. Why might Marta want to reconnect with her religion, and how will attendance at the church camp benefit Marta?

b. If Marta attends the church camp, will this pose a problem to counseling sessions?

Case Study, Mohr

CHAPTER 8,- Nursing Values, Attitudes, and Self-Awareness

In completing the case study, students will be addressing the following learning objective:

Discuss how situational factors can potentially influence the behavior of healthcare professionals toward clients with mental illness.

1. Joe, a 26-year-old Caucasian man, is a client in a state prison system.  Joe is admitted to the prison clinic after being involved in a fight in which he sustained a stab wound to the chest that did not penetrate the lungs or major blood vessels.  The clinic doctor on duty was an employee of several years at the prison.  The doctor showed little compassion for Joe, stating, “He is a convicted criminal, and he is just getting back some of what he deserves.”  The new graduate nurse who was being oriented to the clinic thought that the doctor did not exhibit professional behavior toward Joe.  The clinic nursing supervisor later explained that the doctor was influenced by situational factors.

(Learning Objective: 3)

  • What is a situational factor, and how can this influence the behavior of healthcare workers? Provide an example where a situational factor impacted on your behavior while in a healthcare setting.

Case Study, Mohr

CHAPTER 9, the Nursing Process in Psychiatric–Mental Health Care

In completing the case study, students will be addressing the following learning objective:

Apply the nursing process to psychiatric–mental health nursing.

1. The student nurse is assigned to assist the psychiatric nurse with the admission interview of a client at the psychiatric hospital.  The nurse explains to the student that the interview is very important in obtaining a total health history of the client.  The nurse should be courteous and respectful of the client to obtain as much information from the client as possible.  Assessment information should include the subjective information from the client with the reason for needing treatment, the cause of the present problem(s), and the client’s expectation of the outcome of treatment regimen.  The nurse alerts the student to the need to be sensitive to both verbal and nonverbal behavior of the client and to focus on topics that seem important to the client. Mental Health

(Learning Objective: 1)

a. How can attention to the client’s nonverbal cues be of value in an interview?

b. Why is the interview the most important aspect of the nursing process?

Case Study, Mohr

CHAPTER 10, the Interview and Assessment Process

In completing the case study, students will be addressing the following learning objective:

Describe essential elements to include in psychiatric interviews and the assessment process.

  • The student nurse is accompanying the psychiatric nurse during the nursing interview and assessment of a newly admitted patient.  The psychiatric nurse told the student that preparation with subjective and objective data collection is an important part of the process.  The nurse explains that assessment has reference to the interviewer’s interpretation and prioritization of all data for the client.  The nurse must have self-awareness and self-knowledge to be objective and avoid influencing the responses of the client.  Anxiety on the part of the nurse may limit the ability for thorough data collection and interpretation.  Anxiety in the nurse may evoke anxiety on the part of the client. The psychiatric nurse stressed that a process recording, or written analysis of the interaction between the client and nurse, is essential for nurses to recognize the effects of their communication style in the assessment process.  A review of the client’s history is important, and a private setting for the interview is necessary.  The content of the nursing assessment should include the ability and reliability of the client’s response to questions of the interviewer and the skill of the nurse in identification of relevant facts.  The nurse should discuss with the client prior health history, any present illness, and the reason for seeking healthcare at this time.  Medication history with compliance and allergies of the client require investigation.   Substance use by the client, past illnesses, and family history need exploration.

(Learning Objective: 2)

a   The student nurse asked the clinical psychiatric nurse practitioner to explain what she considered the most essential part of the assessment process. How should the psychiatric nurse respond to the student’s question? Explain your answer.

ND’s pneumonia

Read the case study below. Answer and discuss the questions that follow.

ND is a 82-year-old female who lives in a skilled nursing facility. She suffers from rheumatoid arthritis, coronary artery disease, chronic bronchitis, and hypertension. She is wheelchair bound and tends to spend most of her days in bed. She smoked for 52 years but quit 14 years ago. She has lost 10 pounds in the last month and has had a productive cough for about 2 weeks that has copious amounts of white to yellow mucous. Over the last 3 days, she has refused to get out of bed and has been refusing to eat or drink. This morning she is confused, has a fever, and is coughing continuously. Her sputum now is rust-colored. Her lungs sounds are coarse rhonchi throughout with crackles and diminished lung sounds in her right middle lobe area. Her vital signs are BP 86/54, HR 98, RR 28, and temperature 102.4. She is being admitted to the hospital ND’s pneumonia.

ORDER A FREE-PLAGIARISM PAPER HERE

 

Pneumonia can be classified many different ways. Which classification of pneumonia best fits ND’s situation? Explain. What is the most probable cause of ND’s pneumonia? Which diagnostic tests would you expect to be done to diagnose ND’s pneumonia?

bronchial asthma

Question

Question 1 A patient with bronchial asthma is prescribed a sustained-release preparation of theophylline. To help minimize the adverse effects of the drug, which of the following should the nurse suggest?

Question 2 A female patient has been taking zafirlukast for a week and is experiencing diarrhea. The nurse should instruct her to do which of the following?

Question 3 A 72-year-old man is prescribed theophylline for symptomatic relief of bronchial asthma. Which of the following findings would alert the nurse to the need for close monitoring?

Question 4 A 57-year-old male patient who has been prescribed doxorubicin for small-cell lung cancer is advised by the nurse to avoid taking aspirin or drugs that contain aspirin because it may

Question 5 A 72-year-old female patient is prescribed ipratropium aerosol for pulmonary emphysema. The nurse will instruct her to “test spray” the new metered-dose inhaler (MDI) three times before using it to prevent which of the following?

Question 6 A male patient is to begin treatment for pneumonia with an albuterol (Ventolin) inhaler. The nurse will advise the patient that he will most likely experience which of the following common adverse effects of the drug?bronchial asthma

Question 7 The lower respiratory system utilizes a number of different mechanisms that confer protection and maintain homeostasis. Which of the following physiological processes protects the lower respiratory system?

Question 8 A 70-year-old woman has a complex medical and a current drug regimen that includes calcium and vitamin D supplements for osteoporosis, metformin (Glucophage) for type 2 diabetes, phenelzine (Nardil) for depression, and metoprolol (Lopressor) and furosemide (Lasix) for hypertension. The woman is requesting dextromethorphan for the treatment of a recurrent cough. What component of her drug regimen contraindicates the use of dextromethorphan?

Question 9 A 53-year-old man has been treated for severe asthma for several years with prednisone. Recently, his physician initiated alternate-day therapy for him. The patient tells the nurse that he would rather take the medication every day to prevent confusion. Which of the following would be the best response by the nurse?

Question 10 A nurse is providing discharge planning for a 45-year-old woman who has a prescription for oral albuterol. The nurse will question the patient about her intake of which of the following?

Question 11 A 68-year-old man complains of a chronic, nonproductive cough. He states that he has to have relief, that he has been coughing every 2 to 3 minutes, and he is worn out. Dextromethorphan is prescribed for him. Before he leaves the clinic he asks how long it will take for the medicine to work. The nurse will advise him that he should experience therapeutic effects in

Question 12 A female patient, age 36, is prescribed inhaled corticosteroid (ICS) for daily use. Which of the following adverse effects should the nurse closely monitor for in this patient?

Question 13 A patient is being treated for respiratory infection. He is a recovering alcoholic and has impaired liver function. The nurse will instruct the patient to be especially cautious when taking

Question 14 A college student has presented the campus medical clinic complaining of cold symptoms that he has been experiencing for the past 2 weeks. He tells the nurse that he has been taking OTC decongestants twice daily since the onset of the cold. What teaching point should the nurse provide to this student?

Question 15 A high school student was diagnosed with asthma when he was in elementary school and has become accustomed to carrying and using his “puffers”. In recent months, he has become more involved in sports and has developed a habit of administering albuterol up to 10 times daily. The nurse should teach the student that overuse of albuterol can lead to

Question 16 Which of the following will a nurse inform the patient is one of the most common adverse effects of guaifenesin (Robitussin)?

Question 17 A nurse is assigned to a patient who is receiving IV aminophylline. The nurse is aware that the IV infusion rate should be bronchial asthma

Question 18 A patient with a diagnosis of nonsmall cell lung cancer is currently undergoing chemotherapy. At the encouragement of a family member, the patient has announced to the nurse his intention to complement this treatment with a regimen of herbal remedies. How should the nurse respond to this patient’s statement?

Question 19 A 47-year-old woman has been diagnosed with open-angle glaucoma. Pilocarpine drops are prescribed. The nurse’s assessment reveals that the patient has worn soft contact lenses for 15 years. The nurse will instruct the patient to

Question 20 A man who smokes one pack of cigarettes daily has been diagnosed with chronic bronchitis. He has been prescribed theophylline by his family doctor. What effect will the patient’s smoking status have on the therapeutic use of theophylline?

Question 21 A nurse is providing patient education to a 42-year-old man who has been prescribed dextromethorphan (Robitussin). The patient is a crane operator for a local construction company. The nurse’s assessment reveals that the patient is a smoker and has diabetes. The most important safety consideration while the nurse is designing a plan of care for this patient would be the fact that he is

Question 22 A nurse is caring for a female patient who has developed atelectasis because of thick mucus secretions. The nurse is monitoring IV administration of acetylcysteine (Mucomyst) and notices that the patient’s face is flushed. The appropriate nursing action would be to

ORDER A FREE-PLAGIARISM PAPER HERE

Question 23 A 24-year-old factory worker has been prescribed guaifenesin for the first time. Which of the following will be a priority assessment by the nurse before the patient’s first dose?

Question 24 A 25-year-old woman comes to the clinic because of a chronic, nonproductive cough. Assessment reveals that she has a history of asthma and dextromethorphan, (Robitussin) is prescribed for her. The nurse will question this order because

Question 25 A patient is in the clinic for seasonal allergic rhinitis. Loratadine (Claritin) is prescribed. Which of the following statements will the nurse include when providing patient education concerning this drug?

Question 26 A female patient has been taking prednisone for her asthma for 1 month. The nurse will teach her to gradually decrease her dose of prednisone to avoid

Question 27 A female patient calls the clinic and reports that since she has been taking dextromethorphan (Robitussin), she has been extremely drowsy and dizzy. The nurse will question the patient about which of the following?

Question 28 After a recent history of shortness of breath that has become increasingly severe, a woman has been prescribed ipratropium by MDI while she undergoes a diagnostic workup. What patient teaching should the nurse provide to this patient?

Question 29 The parents of a 7-year-old boy who has just been diagnosed with allergic asthma are being taught about their son’s medication regimen by the nurse. The nurse is currently teaching the parent’s about the appropriate use of a “rescue drug” for acute exacerbations of their son’s asthma. What drug should the nurse suggests the parents to use in these situations?

Question 30 A nurse is providing education to a 56-year-old man who is admitted to the emergency department with an acute asthma attack. The nurse’s initial assessment reveals that the patient has a history of pneumonia, drinks large quantities of coffee, and eats a high-calorie diet.Albuterol is prescribed for him. The important consideration when the nurse is preparing a teaching plan will be that the patient bronchial asthma

Disaster Planning For Public Health

Assignment: Disaster Planning for Public Health

By Day 7 of Week 5

  • Select a potential natural or man-made disaster that could happen in your community. Then, write a 3- to 4-page paper about the disaster from the community nurse’s perspective.

Section 1: The Disaster, Man-Made or Natural

  • What disasters may strike your community and why? For example, do you live in “Tornado Alley,” or has climate change resulted in unusual cold weather snaps or blizzards in your community? Are you located in a flood plain? Include possible diseases that may result from a natural disaster, such as tetanus or cholera Disaster Planning For Public Health.

Section 2: The Nursing Response

  • Formulate responses to the disaster, considering systems and community levels of intervention.
  • Review websites where a disaster plan may be available for the public, or if one is not currently available, call public health department to see if a disaster plan exists for your community and what the plan contains.
  • In addition to reviewing websites for information about your local disaster plan, you will need to locate best practice/evidence-based practice guidelines in professional literature to determine whether your community’s disaster plan is as sound as it might be or if there is room for improvement.

Section 3: Is My Community Prepared for a Disaster?

Week 4: Evidence-Based Practice in Disaster Planning: Nurses as Leaders  

  • Public health surveillance is one way that public health officials target intervention strategies (Turlock, 2016). Often, it is through prompt recognition of and reporting of incidents of communicable disease that a disaster can be averted (Turlock, 2016). Surveillance activities often prompt questions such as, What is causing the disease? How is it spreading? And who is at risk (Turlock, 2016)? While it is true that preparedness planning cannot eliminate all traces of threat to a community, planning assures that medical services and treatment are deployed in an effective, efficient, and rapid manner (Turlock, 2016). Public health plays a vital role in coordination of providers, assurance of supplies particularly when the Strategic National Stockpile pharmaceuticals and supplies are required, and mobilization of state and national response systems. Public health officials may also provide health care services when required (Turlock, 2016).
  • Stanhope (2016) noted that evidence-based practice (EBP) has become more important in health care for many reasons: increased expectations of consumers, increased availability of information through the Internet, increased accountability for results, health care economic changes, and growing numbers of lawsuits, among other reasons. EBP is a lifelong problem-solving approach that regularly produces excellent results and often provides the theoretical underpinnings for programs to mitigate problems in the community. Once programs are in place, evaluation of their effectiveness should be conducted to determine whether they are worth the continued expenditure of resources. Use of EBP is vital to assure safe outcomes for populations during disasters, such as massive communicable disease outbreaks, and should be the foundation of disaster-planning strategies. Disaster Planning For Public Health

Required Readings

Stanhope, M., & Lancaster, J. (2016). Public health nursing: Population-centered health care in the community (9th ed.). St. Louis, MO: Elsevier.

  • Chapter 15, “Evidence-Based Practice” (pp. 342–354)
  • Chapter 23, “Public Health Nursing Practice and the Disaster Management Cycle” (pp. 503–528)
  • Chapter 24, “Public Health Surveillance and Outbreak Investigation” (pp. 529–544)
  • Chapter 25, “Program Management” (pp. 545–567)

Required Media  

  • Laureate Education (Producer). (2009a). Family, community and population-based care: Emergency preparedness and disaster response in community health nursing [Video file]. Baltimore, MD: Author.      
  • TED. (2012). How to step up in the face of disaster [Video file]. Retrieved from                   https://www.ted.com/talks/caitria_and_morgan_o_neill_how_to_step_up_in_the_face_of_disaster
  • This Ted Talk describes the actions of two sisters who step up as leaders during a tornado disaster in their community.

Epidemiology

Question 1 

Suppose that you are following a group of children for the development of asthma over a one- year period. You identify 100 children on January 1st, screen them for asthma, and set up a monitoring program to check on their status on a monthly basis. Five children are considered prevalent cases because they were diagnosed with asthma before January 1st. Ten children develop asthma on March 1st and another ten children develop asthma on July 1st. Another 10 children who remain healthy were followed for six months and then were lost to follow-up. All of the remaining children did not develop asthma and were not lost to follow-up. Follow-up ended on December 31st.

· What was the prevalence of asthma on June 1st?

· What was the prevalence of asthma on September 1st?

· How many person-months of observation were accrued by this population?

· What was the incidence rate of asthma in this population for the one-year period?

ORDER A FREE-PLAGIARISM PAPER HERE

Question 2 

A population of 1,000 people is monitored for a year for the development of measles. No one has measles at the start of the investigation. Thirty people develop measles on June 30 and twenty people develop measles on September 30. Eight people are lost to follow-up on March 31 and twenty-four people are lost to follow-up on November 30. None of those lost to follow-up had developed measles prior to becoming lost. Assume that you can only get measles once.

· What is the cumulative incidence of measles in this population?

· What is the incidence rate of measles?

· What is the prevalence of measles on July 1? Epidemiology

Question 3 

· Which type of measure of disease frequency best describes each of the following scenarios?

· Percentage of students enrolled in a college who developed influenza during the spring semester of 2012. 

· Percentage of students enrolled in an epidemiology class who had sore throats on the first day of class.

· Percent of breast cancer patients who underwent mastectomy during 2012.

· Percent of men found to have high blood pressure at their yearly physical.

· Number of newly-diagnosed cases of AIDS in a year per 100,000 persons.

· Percent of infants born with spina bifida out of 1,000 live-born infants.

· Percent of drivers found to be legally drunk at the time of their car accident.

Psychiatric Nursing

STUDY GUIDE

This quiz will cover the following topics, which relate to psychotherapy with individuals in the Wheeler textbook and the Fisher textbook excerpts:

Topics

1. The Nurse Psychotherapist and a Framework for Practice

1. Confidentiality Limits in Psychotherapy: Ethics Checklists for Mental Health Professionals

1. Assessment and Diagnosis

1. The Initial Contact and Maintaining the Frame

1. Cognitive Behavioral Therapy

1. Supportive and Psychodynamic Psychotherapy

1. Stabilization for Trauma and Dissociation

1. Motivational Interviewing

1. Psychotherapeutic Approaches for Addictions and Related Disorders

Wheeler, K. (Eds.). (2014). Psychotherapy for the advanced practice psychiatric nurse: A how-to guide for evidence-based practice(2nd ed.). New York, NY: Springer Publishing Company.

1. Chapter 1, “The Nurse Psychotherapist and a Framework for Practice”

1. Chapter 3, “Assessment and Diagnosis”

1. Chapter 4, “The Initial Contact and Maintaining the Frame”

1. Chapter 5, “Supportive and Psychodynamic Psychotherapy”

1. Chapter 7, “Motivational Interviewing” (pp.299-312)

1. Chapter 8, “Cognitive Behavioral Therapy” Psychiatric Nursing

1. Chapter 13, “Stabilization for Trauma and Dissociation”

1. Chapter 16, “Psychotherapeutic Approaches for Addictions and Related Disorders”

Fisher, M. A. (2016). Introduction. In Confidentiality limits in psychotherapy: Ethics checklists for mental health professionals (pp. 3–12). Washington, DC: American Psychological Association. doi:10.1037/14860-001

Fisher, M. A. (2016). The ethical ABCs of conditional confidentiality. In Confidentiality limits in psychotherapy: Ethics checklists for mental health professionals (pp. 13–25). Washington, DC: American Psychological Association. doi:10.1037/14860-002

Question 1

1. After informing a prospective patient about limits of confidentiality, the patient consents to the “conditions” of confidentiality and signs an informed consent form. Several weeks later, a lawyer representing the patient’s spouse for a court case, asks the PMHNP for the disclosure of information about the patient. The PMHNP should:

  A. Disclose all patient information as requested
  B. Refuse to disclose any confidential information
  C. Limit disclosure to the extent legally possible
  D. Ignore the request based on ethical reasons

1 points  

Question 2

1. The PMHNP is meeting with a new patient who is a young veteran back from serving two tours overseas. When reviewing the patient’s health history file that was sent from the VA, the PMHNP learns that several months ago, the patient was diagnosed with PTSD, but never followed up with treatment for it. After a comprehensive mental health assessment, the PMHNP confirms the PTSD diagnosis and understands that which therapeutic approach will be the most effective as a first-line treatment modality?

  A. Performing a psychiatric debriefing on the patient
  B. Employing cognitive behavioral therapy with the patient
  C. Using psychodynamic psychotherapy with the patient
  D. None of the above

1 points  

Question 3

1. One of the strategies the PMHNP wants to try includes sleep restriction. What will the PMHNP suggest to follow sleep restriction therapy?

  A. “Restrict sleep for 24 hours.”
  B. “Restrict the amount of time you spend in bed.”
  C. “Restrict the amount of exercise you do prior to going to bed.”
  D. “Restrict the amount of food you eat before bedtime.”

1 points  

Question 4

1. The PMHNP is conducting a peer review of another PMHNP’s medical charts. Upon review, the PMHNP notes that the peer often begins patient sessions late, as well as ends them later than scheduled. The PMHNP also found a comment in the chart regarding the patient sending text messages while in the middle of the session. Based on these findings, the feedback that the PMHNP will provide to the peer involves which therapeutic principle?

  A. Assessing safety
  B. Applying therapeutic communication
  C. Using empathy
  D. Maintaining the frame   32.The PMHNP is mentoring a student. After working with a patient during a session, the student laments about all the things she should have, or could have, said to the patient. “I feel guilty that I didn’t speak up more about the patient’s concern toward her son,” says the student. The PMHNP understands that the student is exhibiting signs of which therapeutic concept?

1 points  

Question 5

1. True or false: If after a third round of processing followed by stabilization, a patient has increased anxiety that requires further interventions aimed at stabilization, the therapeutic relationship can no longer successfully provide therapeutic change.

True

False

1 points  

Question 6

1. The PMHNP has been treating a 15-year-old patient with a history of abuse and neglect. Thirty minutes into their therapy session the patient jumps up and begins to pace around the room. Utilizing Socratic dialogue (SD) the PMHNP’s best action would be to:

  A. Allow the patient time to process before speaking.
  B. Ask, “Why don’t you come have a seat beside me?” while tapping the table.
  C. Say, “I noticed a change. Can you tell me what happened?”
  D. Say, “We can continue this session later if you prefer.” 

1 points  

Question 7

1. The PMHNP is caring for a patient who the PMHNP believes would benefit from a relational psychodynamic approach to therapy. Which action made by the PMHNP demonstrates appropriate use and understanding of the relationship psychodynamic model?

  A. Focusing the exploration on making the unconscious conscious
  B. Focusing the exploration on the genetic roots of the patient’s problem
  C. Focusing the exploration on here and now
  D. None of the above

1 points  

Question 8

1. The PMHNP is caring for a patient with borderline personality disorder. Using a psychoanalytic psychotherapy approach, the PMHNP attempts to intensify the patient’s transference to enhance emotional processing by:

  A. Developing increased boundaries
  B. Decreasing supportive psychotherapy
  C. Scaling back contact with the patient
  D. Increasing the number of sessions per week

1 points  

Question 9

1. An elderly patient in a nursing home has been losing interest in activities and now refuses to leave his room. After a physical exam, he is referred to a PMHNP for an initial assessment. True or false: After speaking with the patient, an appropriate screening tool for the PMHNP  to use would be the Geriatric Depression Scale. Psychiatric Nursing

True

False

1 points  

Question 10

1. A middle-aged man who works over 50 hours a week is being seen for depression and anger management. He states, “I am even more frustrated when I come home and my wife wants to argue about stupid stuff. All I want to do is come home, take a shower, and eat. Is that too much to ask?” The PMHNP explains that people can be assertive, aggressive, and passive. She encourages the patient to be more assertive and begins role-play with assertive training. The PMHNP determines that the patient is beginning to understand when he states:

  A. “I am tired, I work very hard all day to support you and this family!”
  B. “I come home and all you do is argue, but I don’t care.”
  C. “I don’t know what I did. Why are you acting like this?!”
  D. “I feel frustrated when I come home and we argue.” 

1 points  

Question 11

1. The PMHNP is treating a patient with a substantial fear of feeling closed in (claustrophobia). Thus, the patient will not get into an elevator. The office where he works is on the 10th floor and this requires that he walk up and down the stairs in the morning and evening to get to his office. With permission from the patient, the PMHNP is beginning systematic desensitization to address the patient’s need to use the elevator. What is the PMHNP’s best plan of action?

  A. Begin by having the patient stand in front of the elevator and write down his feelings
  B. Complete a 30-minute therapy session in an elevator
  C. Allow the patient to watch an elevator go up and down
  D. Allow the patient to see the PMHNP getting into an elevator

1 points  

Question 12

1. A PMHNP is assessing ego functioning of his 40-year-old patient by asking what she feels is the cause of her problems. She attributes her problems to her overprotective parents not letting her have enough freedom growing up. Based on her answer, the PMHNP is testing _____________.

  A. adaptive regression in the service of the ego
  B. regulation and control of affects and impulses
  C. defensive and interpersonal functioning
  D. sense of reality of the world and of the self

1 points  

Question 13

1. The PMHNP meets with a 47-year-old male patient who is fearful of leaving the house after having witnessed his neighbor getting run over by a car. When the PMHNP asks why he is afraid to leave his house, the patient replies, “Because another accident might occur.” Which cognitive behavioral therapy (CBT) strategy does the PMHNP employ?

  A. Pharmacological therapy
  B. Stress inoculation therapy
  C. Dialectical behavior therapy
  D. All of the above

1 points  

Question 14

1. A 62-year-old patient has been diagnosed with borderline personality disorder. Upon assessment, the PMHNP learns that he  participates in spending sprees and occasional binge eating, accompanied by rapid changes in self-image. Which evidence-based psychotherapeutic model does the PMHNP identify as effective and beneficial?

  A. Cognitive behavioral therapy
  B. Supportive therapy
  C. Dialectical behavior therapy
  D. Expressive therapy

1 points  

Question 15

1. The PMHNP is working with a patient who witnessed her father pass away after suffering for several months from terminal cancer. The PMHNP sees this as a traumatic event. The patient reports sometimes feeling out of touch with surroundings; almost as if things feel like a dream. “Sometimes that sensation lingers for a while,” the patient says, “and other times I snap out of it quickly.” What does the PMHNP infer about the condition based on psychotherapy concepts for trauma?

  A. The patient is having a balanced response to the trauma.
  B. The patient is reporting signs of dissociation.
  C. The patient is becoming unresponsive to the environment.
  D. All of the above.

1 points  

Question 16

1. The patient is a 56-year-old female patient diagnosed with panic disorder and reports symptoms that include heart palpitations, frequent trembling, and feelings of choking in stressful situations. What special consideration does the PMHNP make?

  A. Using a special focus on transference as the agent promoting change
  B. Confronting the emotional significance of the patient’s symptoms
  C. Employing psychodynamic therapy combined with pharmacotherapy
  D. A and B

1 points  

Question 17

1. The PMHNP is meeting with a 38-year-old divorced, single mother who has been seeing the PMHNP for anxiety and anger management. During one of the sessions, the patient tells the PMHNP that she is having a problem getting her child support payments from her ex-husband, which is triggering increased anxiety and anger, which she admittedly takes out on her children. How does the PMHNP respond in a way that upholds the supportive psychodynamic psychotherapy approach? Psychiatric Nursing

  A. Telling the patient to take a vacation to get away and relax
  B. Suggesting that she seek legal help
  C. Giving her advice about men and how to get them to pay child support
  D. Suggesting that the patient start going to church to help find more solace.

1 points  

Question 18

1. During a session, the PMHNP asks a patient with a history of sexual abuse to recall the relationship she had as a child with her parents. The patient responds by saying she has a headache and her stomach hurts, and starts talking about her physical ailments. What is the appropriate response by the PMHNP?

  A. Assure the patient that she does not have to discuss these memories
  B. Explain why it is important for the patient to discuss these memories openly
  C. Tell the patient that she may not get better if she does not face her fears
  D. Let the patient know that plenty of other people have similar painful memories, too

1 points  

Question 19

1. The PMHNP is working with an adult patient who has somatic complaints caused by a history of childhood abuse. As part of the therapeutic process, what does the PMHNP do to assess and organize a trauma history?

  A. Interview the patient’s family members
  B. Construct a timeline of the patient’s life
  C. Review the patient’s current stressors
  D. Debrief with the patient

1 points  

Question 20

1. During a therapy session with an 18-year-old female the PMHNP learns that she has lived in six different foster homes in the last 24 months. She states that her mother is in a correctional facility for drug abuse and prostitution. During the assessment, the PMHNP notices multiple superficial cuts to each wrist. The patient appears tearful, withdrawn, and never makes direct eye contact. The PMHNP believes that this patient may have feelings of insecurities as well as abandonment issues and is aware that which diagnosis is likely possible?

  A. Narcissistic personality disorder
  B. Obsessive-compulsive personality disorder
  C. Borderline personality disorder
  D. Paranoid personality disorder

1 points  

Question 21

1. A patient named Steve is seeking therapy to get help with his home situation, stating that he has been stressed since his mother-in-law moved into the house. What can the PMHNP do to assist the patient in constructing a narrative?

  A. Share a personal story about her home situation and family members
  B. Elicit details so Steve becomes more self-disclosing and self-examining
  C. Guide Steve through imagery exercises so he can decrease his stress
  D. None of the above

1 points  

Question 22

1. A 16-year-old female patient has had a stable therapeutic relationship with the PMHNP for several months. The PMHNP notices what appears to be fresh cigarette burns on the patient’s arm. When asked, the patient admits to this self-injurious behavior. Although the patient is still depressed, her mood seems to be somewhat better this week. True or false: The PMHNP would most likely consider this patient stabilized and ready to move to Stage II.

True

False

1 points  

Question 23

1. The PMHNP is working with a patient who experiences abreactions when discussing repressed feelings of his sexual abuse as a child. What can the PMHNP do to manage the patient’s intense emotional reactions?

  A. Sit closer to the patient so the patient does not feel isolated
  B. Embrace the patient to provide physical comfort
  C. Suggest a relaxation technique, such as yoga or meditation
  D. All of the above

1 points  

Question 24

1. A PMHNP has been working with a young female patient who suffers from depression to change self-defeating behaviors Psychiatric Nursing. By creating a presence of acceptance and using good listening skills, the PMHNP’s overall goal is to __________.

  A. deepen the patient’s understanding of herself in order to cultivate empowerment
  B. slowly transfer authority to the patient when the PMHNP feels that she is ready
  C. remain caring, yet authoritative by making important decisions for the patient
  D. all of the above

1 points  

Question 25

1. During cognitive behavioral therapy (CBT), a 64-year-old male patient states, “I get so frustrated sometimes and I just blow up at everybody!” Which response by the PMHNP demonstrates translation of Socratic dialogue (SD)?

  A. “Please explain how this affects the relationship with your wife.”
  B. “When you say blow up, what exactly does blow up mean and  how does it feel to you?”
  C. “What coping methods have you used in the past during times of frustration?”
  D. “So you blow up when you become frustrated?”

1 points  

Question 26

1. The PMHNP is mentoring a student. After working with a patient during a session, the student laments about all the things she should have, or could have, said to the patient. “I feel guilty that I didn’t speak up more about the patient’s concern toward her son,” says the student. The PMHNP understands that the student is exhibiting signs of which therapeutic concept?

  A. Countertransference
  B. Therapeutic communication
  C. Empathy
  D. Boundaries

1 points  

Question 27

1. A 28-year-old male patient is experiencing distress related to the workplace. What might the PMHNP ask to assess affective development?

  A. “How do you think your colleagues feel about you?”
  B. “How do you calm yourself when you are upset at the office?”
  C. “How do you think your colleague felt when you blamed her?”
  D. All of the above.

1 points  

Question 28

1. A PMHNP is preparing confidentiality forms for his patients. What is the most appropriate first step he should take?

  A. Talk to potential patients to get their feedback and suggestions
  B. Research his profession’s ethics code and state/federal laws
  C. Use current patient interactions to decide what matters most
  D. All of the above

1 points  

Question 29

1. A PMHNP is caring for a 21-year-old woman who is suffering from anxiety and is having difficulty managing stress. Which of the following would be an appropriate step by the PMHNP?

  A. Asking the patient what she does to relieve anxiety
  B. Giving the patient basic stress management activities
  C. Developing a plan together that is not overwhelming
  D. All of the above

1 points  

Question 30

1. The PMHNP is caring for a patient who has acute stress disorder after experiencing a traumatic event 1 week prior. The PMHNP wants to begin with a therapeutic framework that follows the adaptive information processing (AIP) model. What is the priority action for the PMHNP to take?

  A. Focus on the patient’s safety and stabilization
  B. Help the patient process painful memories
  C. Encourage the patient to find ways to achieve personal growth
  D. Assist the patient in planning for the future

1 points  

Question 31

1. The PMHNP is working with a 43-year-old male patient who develops idiosyncratic transference over weeks of therapy. The best way for the PMHNP to respond to this patient is to ___________:

  A. employ cognitive behavioral therapy
  B. use empathy
  C. utilize intersubjectivity
  D. none of the above

1 points  

Question 32

1. The PMHNP is working with a 56-year-old man who is being seen because of his anxiety and depression. During the therapy sessions, the PMHNP assists the patient in discussing his experiences and expanding on his thoughts and feelings. He tells the PMHNP stories about how he used to be teased by his coworkers at the office for becoming too anxious in large conference rooms. The PMHNP listens to the patient and helps focus on his strengths to promote self-understanding. Which principle best accounts for the PMHNP’s interactions with the patient?

  A. Therapeutic communication
  B. Maintaining the frame
  C. Working with resistance
  D. None of the above

1 points  

Question 33

1. During a therapy session the patient is asked how she currently deals with stress, and she says, “Well, I don’t. I just let it build and build.” The PMHNP responds by asking how well this has been working out for her. The patient states, “Well, to be honest, it just feels like I’m drowning, you know what I mean.” Illustrating idiosyncratic meaning, the PMHNP responds with: Psychiatric Nursing

  A. “Would you say you feel like you are suffocating?”
  B. “No, I am not sure what you are saying. Can you please explain?”
  C. “Sort of like a fish out of water?”
  D. “When you say, ‘I just can’t breathe,’ you do understand, that is a form of anxiety?” 

1 points

ORDER A FREE-PLAGIARISM PAPER HERE

Question 34

1. The PMHNP is having a therapy session with Charlotte, a 20-year-old victim of date rape. The patient states, “I shouldn’t have been there; I should have just gone home. This was all my fault; how could I have been so stupid?” Using the Socratic method, what is the PMHNP’s best response?

  A. “If this had happened to someone else with the exact same circumstances, would you say this was her fault?”
  B. “Can you recall exactly what stopped you from going home when you originally wanted to go home?”
  C. ”Have you shared these thoughts about what you should have done with anybody other than me?”
  D. “Do you understand that there is absolutely nothing that you could ever do to deserve to be treated the way you were treated?”

1 points  

Question 35

1. At the initial interview with a patient, the PMHNP reviews the condition of receiving services, including limits that will be imposed on confidentiality. During the discussion, the patient shares information that the PMHNP is legally required to report. True or false: If the PMHNP does not report information that s/he is legally required to report, state laws govern the consequences which include penalties for not reporting, especially child and elder abuse.

True

False

1 points  

Question 36

1. An initial evaluation reveals that an 11-year-old patient has moved to a new school after her parents’ recent divorce, and is having trouble making friends. The patient has normal mental status and exhibits appropriate behavior. What is the most appropriate scale for the PMHNP  to use to get more information?

  A. Young Mania Rating Scale
  B. Hamilton Anxiety Rating Scale
  C. Dissociative Experiences Scale
  D. Impact of Event Scale

1 points  

Question 37

1. The PMHNP is meeting with a 42-year-old man with depression brought on by the recent passing of his wife. As he describes the circumstances surrounding his late wife’s death, the PMHNP begins to feel sad. The sadness lingers for several hours, and the PMHNP finds it difficult to focus on other patients for the rest of the day. What is the most appropriate explanation for the reaction that the PMHNP is experiencing?

  A. Autognosis
  B. Complementary identification
  C. Concordant identification
  D. Self-disclosure

1 points  

Question 38

1. The PMHNP employs psychodynamic psychotherapy with a patient who experiences anxiety and depression. As the process enters the psychoanalytic end of the psychodynamic continuum, the PMNHP will focus on:

  A. Interpreting unconscious conflict in the patient
  B. Restoring the patient’s functioning and stabilization
  C. Reducing the patient’s anxiety
  D. Strengthening the patient’s defenses

1 points  

Question 39

1. The PMHNP is working with a patient who experiences anxiety around her parents that later leads to poor impulse control. What will the PMHNP do to employ psychodynamic psychotherapy properly for this patient?

  A. Emphasize the past
  B. Focus on expressions of emotion
  C. Identify patterns in relationships
  D. All of the above

1 points  

Question 40

1. A new patient has been informed of the limits of confidentiality, and has signed informed consent forms. No consent, however, has been obtained for voluntary “Release of Information.” The patient is 20 years old and still lives at home with his parents. He is being treated for depression, which he attributes to the trouble he’s had finding employment.  True or false: If the patient’s mother calls the PMHNP to check up on her son to see how he is doing, the therapist is required to protect patient confidentiality and not disclose information.

True

False

1 points  

Question 41

1. A PMHNP is treating a 45-year-old female patient who is upset that her brother has not been calling since his divorce. When asked to describe her brother and what prompts him not to call as frequently, she says, “Everything is about him, not me. I think he’s jealous that I have a good marriage and he wants me to be unhappy, too. So he doesn’t call to upset me and ruin my relationships.” Based on this information, the PMHNP can conclude that the patient _________.

  A. may lack ability to see independent motivations
  B. has an insistent emphasis on the feelings of others
  C. has an intrusive interpersonal relationship
  D. all of the above

1 points  

Question 42

1. The PMHNP is assessing a patient who grew up in a foster home because she was neglected and abused by her birth parents at a young age. The patient admits to having difficulty forming and maintaining relationships throughout her life. Understanding maladaptive schemas, which statement does the PMHNP predict that the patient is likely to make?

  A. “I deserve the utmost respect from everyone who meets me.”
  B. “I prefer doing everything on my own.”
  C. “It takes me a while to warm up to people; people often wear masks.”
  D. “I can’t seem to do anything on my own.” 

1 points  

Question 43

1. A patient who has borderline personality disorder is meeting with the PMHNP. When asked about future goals, the patient responds, “I’d like to go back to school to do what you do. You know, talk to people all day about their problems. It seems pretty easy.” How does the PHMNP respond to the client in a way that is free from any stigmatizing beliefs or judgments?

  A. “It is dangerous to fantasize about the future.”
  B. “I think that’s an excellent idea! I can help you review the prerequisites!”
  C. “You may want to explore the requirements for becoming a PMNHP.”
  D. None of the above. Psychiatric Nursing

1 points  

Question 44

1. True or false: A PMHNP may ask his patient to describe her relationship with her father, both as a child and now, in order to assess interpersonal style.

True

False

1 points  

Question 45

1. A 41-year-old male patient is meeting with the PMHNP and reveals that he is homosexual. He begins telling the PMHNP about his feelings as a homosexual, middle-aged man. The PMHNP nods understandingly. Before long, the patient asks, “Are you gay? Are you married? Do you have kids?” What is the best response by the PMHNP?

  A. Answer the patient’s questions honestly to establish trust
  B. Mention that the patient seems quite curious, and ask him to discuss more about himself
  C. Ignore the patient’s questions and move on to another topic
  D. None of the above

1 points  

Question 46

1. The PMHNP is working with a veteran who has posttraumatic stress disorder (PTSD). The PMHNP believes that dual awareness will be beneficial in allowing the patient to focus on the here and now. What strategies can the PMHNP use to develop dual awareness in the patient?

  A. Asking the patient to recall a recent and mildly disturbing event
  B. Having the patient focus on details of the room, such as how hot or cold it is
  C. Telling the patient to rate the level of disturbance he feels from a mildly disturbing event
  D. All of the above

1 points  

Question 47

1. The PMHNP has been assisting a shy and timid 23-year-old male patient struggling with esteem and self-concept issues. Over the weekend, this patient was given the task of completing an exercise that would usually cause him distress and shame, also called a shame-attacking exercise. The PMHNP determines that this task has successfully been achieved when the patient states:

  A. “I was able to eat in public yesterday despite how loudly I chew.”
  B. “I confessed all of my infidelities to my ex-spouse yesterday.”
  C. “I calmly sat through a conversation with my spouse in which she confessed all of her infidelities to me.”
  D. “I journaled the shame and guilt that I’ve been feeling from my past that has been holding me back.” 

1 points  

Question 48

1. A 35-year-old patient seeks treatment for depression and anxiety after an abusive relationship. To help empower the patient, the PMHNP wants to teach the safe-place exercise to create a feeling of calm. In order to walk the patient through the exercise, the PMHNP first says:

  A. “Identify an image of a safe place that makes you feel calm.”
  B. “Think about the things that cause you anxiety and let them go.”
  C. “Take a deep breath and start to relax with each new breath.”
  D. “Picture a beautiful beach and describe to me what you see.”

1 points  

Question 49

1. When conducting an initial assessment, a PMHNP has finished conducting a mental status examination with 10 minutes left in the session. The patient is distressed and would like to continue sharing her personal history. True or false: An appropriate response that the PMHNP might tell the patient is, “I understand that you are upset. [Summarizes diagnosis, treatment recommendations, and follow-up plan.] We can continue the conversation next week. How does this time next Monday work for you?”

True

False

1 points  

Question 50

1. A PMHNP’s patients have already signed confidentiality agreements, including limits that will be imposed on confidentiality. Months later, the state’s laws have changed that affect confidentiality promises the PMHNP has already made. What is an appropriate step for the PMHNP to take after finding out this information?

  A. Revise intake forms to align with state laws
  B. Review confidentiality revisions with patients
  C. Have patients give informed consent again
  D. All of the above

1 points  

Question 51

1. A 65-year-old patient has suffered the loss of his wife. He is in a state of hyperarousal with increased sympathetic nervous system arousal. One or more interventions may help the patient to deal with this arousal. To decrease sympathetic nervous system arousal, the PMHNP’s treatment strategy is ___________.

  A. mindfulness techniques
  B. deep breathing exercises
  C. self-regulation strategies
  D. all of the above

1 points  

Question 52

1. A PMHNP is treating a 50-year-old patient who suffers from depression. When he was growing up, he was often responsible for taking care of his three younger siblings. Since then, the patient puts other people’s needs before his own. Based on this information, the PMHNP would conclude that his interpersonal style is __________.

  A. overly nurturing
  B. nonassertive
  C. exploitable
  D. vindictive

1 points  

Question 53

1. The PMHNP meets with an adolescent patient who has depression and often presents with resistance when discussing his parent’s divorce. For the past couple of sessions, the patient has been quiet, sometimes refusing to speak. To further support the therapeutic relationship, the PMNHP:

  A. Observes and points out the behavior
  B. Stays quiet until the patient is ready to speak
  C. Brings in a colleague to help get the patient to talk
  D. None of the above

1 points  

Question 54

1. The PMHNP has a session with a patient who was injured and traumatized from an explosion at the industrial plant where he used to work. During the session, an outside noise startles the patient and he experiences a flashback of the loud boom from the explosion. The PMHNP witnesses the patient present with anxiety and belabored breathing. What is the appropriate action of the PMHNP?

  A. Telling the patient there is no reason to be startled
  B. Avoiding interrupting the flashback as it occurs
  C. Asking the patient to describe the flashbacks
  D. Have the patient use the relaxation response (take a deep breath, exhale long, and slow down)

1 points  

Question 55

1. In your office, you see a 58-year-old patient with PTSD who reports having nightmares, making it impossible for her to sleep Psychiatric Nursing. What recommendations based on principles of sleep hygiene will the PMHNP make?

  A. Eat a heavy meal before bedtime to induce sleepiness
  B. Maintain a warm bedroom temperature
  C. Avoid watching television or reading in bed
  D. Try going to sleep at different times throughout the week to avoid monotony 

1 points  

Question 56

1. A 35-year-old patient has been seeking treatment for depression for several months. The PMHNP does an assessment to see if processing has led to adaptive change. The patient’s self-references are positive in relation to past events, work is productive, relationships are adaptive, and there is congruence between behavior, thoughts, and affect. Based on this information, the PMHNP decides the patient ____________.

  A. will require more processing strategies
  B. would benefit from the stabilization stage
  C. is ready to move to Stage III, future visioning
  D. no longer needs to continue with therapy

1 points  

Question 57

1. The PMHNP is working with a patient who is living in poverty and abusing substances. According to the treatment hierarchy framework, what needs does the PMHNP need to attend to first?

  A. Provide safety through case management strategies
  B. Help the patient manage positive and negative emotions
  C. Explore the meaning of significant adverse life experiences
  D. Process all dimensions of memory associated with the trauma

1 points  

Question 58

1. Your patient is a 65-year-old male who has a strained relationship with his son and daughter. His children refuse to participate in a family session. The PMHNP asks the patient to draw his family genogram as a next step to _______________.

  A. assess the patient’s mental health status and functioning
  B. explore the interpersonal styles of each family member
  C. provide background information for the patient’s family structure
  D. measure the patient’s quality of well-being and productivity

1 points  

Question 59

1. During cognitive behavioral therapy (CBT), a 64-year-old male patient, states, “My wife hates me! She’s just waiting for me to die.” Using Socratic dialogue (SD) the PMHNP demonstrates understanding of analysis when she responds:

  A. “Now, I’m sure your wife doesn’t hate you.”
  B. “Where is the evidence that your wife hates you?”
  C. “You seem convinced that your wife hates you.”
  D. “What has your wife done to make you believe this?” 

1 points  

Question 60

1. The PMHNP is working with a patient who has insomnia and battles going to sleep. The patient reports consistent self-defeating behaviors, and hates trying to go to sleep. Which action made by the PMHNP demonstrates the use of a paradoxical intervention?

  A. Telling the patient to start trying to go to sleep earlier, since it takes long to fall asleep
  B. Telling the patient to resist sleep and stay awake
  C. Having the patient perform a relaxing ritual before bedtime
  D. Instructing the patient to avoid stimulants and sugars for 3 hours before falling asleep

1 points  

Question 61

1. The PMHNP is working with a patient who seems dissatisfied with the therapeutic relationship. The PMHNP invites the patient to discuss her feelings regarding the PMHNP openly and honestly. It becomes clear to the PMHNP that they are experiencing an alliance rupture. How does the PMHNP repair the therapeutic alliance?

  A. Responding to the patient in a nondefensive manner and accepting responsibility for the PMHNP’s part in the tension
  B. Emphasizing with the patient’s experiences and validating the patient for bringing it up
  C. Considering changing the goals of the patient’s treatment
  D. All of the above

1 points  

Question 62

1. The patient attempts the PMHNP’s sleep hygiene recommendations for 2 weeks, but does not make any progress mitigating nightmares and hyperarousal. Which behavioral strategy does the PMHNP suggest next?

  A. Progressive muscle relaxation exercises
  B. Paradoxical interventions
  C. Biofeedback
  D. All of the above

1 points  

Question 63

1. A PMHNP is taking a history and wants to assess how the patient copes with adversity. To do so, the therapist asks questions about the patient’s belief system. What would be an appropriate question to ask?

  A. “What are your beliefs about therapy and seeking help?”
  B. “To what extent do your religious beliefs give you comfort?”
  C. “What gives you a sense of meaning and purpose in life?”
  D. All of the above.

1 points  

Question 64

1. An 11-year-old patient has been exhibiting low self-esteem at school and acting out. According to Maslow’s hierarchy of needs, which of the following questions would best be addressed first by the PMHNP?

  A. “Are Joel’s peers including him at recess?”
  B. “Does Joel feel safe and secure at school?”
  C. “Is Joel getting enough sleep at home?”
  D. “How can Joel feel that he is loved?”

1 points  

Question 65

1. The PMHNP is treating a 35-year-old male officer in the military. He discloses that both of his parents are deceased and that he loved them. However, he says that he had feelings of inadequacy because his parents held him to a standard that he could never achieve. He went on to say that nothing he did ever felt good enough. The PMHNP assesses that this patient has perfect creases in his uniform with no strings or tags out of place; she also notices that he has perfect posture and questions him about ritualistic behaviors. She suspects that this patient has maladaptive responses to the expectations placed on him as a teenager and young adult. Which statements made by the patient would verify the PMHNP’s suspicion? Psychiatric Nursing

  A. “I typically don’t listen to anyone. I take care of my own wants and needs so I feel like no one can judge me or criticize me, period.”
  B. “I don’t mean to hurt other people’s feelings. When people cry or say that I have made them upset in some way, that’s not my fault; some people are just sensitive.”
  C. “I like to listento the beat of my own drum; I don’t mind spending most of my days alone. I don’t  need recognition or praise; I would just like to be left alone.”
  D. “I believe in systems; I have to have order and rules in my everyday life. If a task must be completed, I will often complete it myself versus depending on someone else.” 

1 points  

Question 66

1. The PMHNP is caring for a patient who seems to seek affection and attention from the PMHNP and others in the clinic, as well as displays heightened emotional responses to feelings of being excluded. What therapeutic approach does the PMHNP use to decrease autonomic arousal in the patient?

  A. Group therapy
  B. Controlled confrontation
  C. A safe-place exercise
  D. Body and energy work

1 points  

Question 67

1. The PMHNP is caring for a patient with dissociated self-state that the PMHNP identifies as being associated with traumatic experiences in the patient’s past. What approach does the PMHNP use with the patient that is crucial to the psychodynamic therapy process?

  A. Assisting the patient to experience and accept the various dimensions of the self through enhanced awareness of the traumatic states
  B. Becoming a co-participant in the co-construction of the relationship with the patient, rather than an outside observer
  C. Making associations between an event or situation and the patient’s feelings
  D. Providing empathy, understanding, and soothing to help the patient identify the other self-states

1 points  

Question 68

1. The PMHNP understands that anxiety and depression are two disorders in which their symptoms may overlap. When discussing this, you explain that the autonomic nervous system is activated and further helps distinguish anxiety symptoms by making the following points:

  A. “You may experience an elevated heart rate, constant worrying, and diarrhea.”
  B. “You may experience fear, sweating, and muscle tension.”
  C. “You may experience an elevated heart rate, the need to urinate, and fear.”
  D. “You may experience sleepiness, decreased blood pressure, and fear.” 

1 points  

Question 69

1. The PMHNP is working with a patient who has dissociative disorder and requests pharmacological interventions for dealing with her trauma. What education does the PMHNP provide to the patient regarding medication therapy?

  A. “The medication will help you forget the things that trigger your flashbacks.”
  B. “The medication takes a while to build up in your system and cure you.”
  C. “The medication may provide symptom relief, but you still need psychotherapy.”
  D. All of the above.

1 points  

Question 70

1. When a PMHNP is seeing a patient for the first time, what is an important step to assure that the patient and provider understand the limits of their discussion?

  A. Assure the patient that she is safe to discuss her secrets
  B. Talk about the importance of being truthful and open
  C. Have a discussion about the confidentiality and its limits
  D. Mention the possible need for selective self-disclosure

1 points  

Question 71

1. During a clinical assessment of a 15-year-old patient, the PMHNP asks, “How can I help you?” The patient answers by saying, “Honestly, I don’t really think I need any help.” Which of the following is the most appropriate response by the PMHNP?

  A. “That’s fine. Can you describe the depressive symptoms you’ve been having the past few months?”
  B. “Since you’re already here, maybe we can try to figure out if there is anything else I can help you with.”
  C. “You wouldn’t be here if you didn’t actually need help, right? So tell me, how can I be of assistance?”
  D. None of the above.

1 points  

Question 72

1. You are seeing a 29-year-old widow whose husband recently died overseas while serving his country in the military. She has been mourning the loss of her husband for several months, and continues to grieve. She refuses to go to group grieving sessions, but reports that she is still able to go to work and her fitness classes sometimes, and even makes attempts to stay social. She says, “Sometimes it’s like he’s not even gone. Other times it feels like it’s been an eternity since I’ve seen him. It’s hard to talk about this type of stuff with my girlfriends, especially since all of their husbands are still alive.” The PMHNP understands that it is appropriate to employ which therapeutic principle?

  A. Encouraging catharsis
  B. Encouraging abreaction to repressed feelings
  C. Identifying conflict-resolution techniques
  D. All of the above

1 points  

Question 73

1. When the patient comes into the office, she says, “I just saw a friend of mine out in the waiting room. What’s wrong with him?” The PMHNP says, “He’ll be fine. He has mild depression.” Which of the following statement is correct related to confidentiality rights?

  A. The PMHNP was not protecting patient confidentially rights.
  B. The PMHNP was not using identifying information in the patient situation above.
  C. Because the patient signed a consent form, the PMHNP was legally allowed to share information.
  D. The PMHNP’s response was ethical and legal because she was trying to answer the question.

1 points  

Question 74

1. The PMHNP is working with a school-aged child who has been diagnosed with depression. The child has attended several sessions with the PMNHP, but recently presents with avoidant behavior by showing increased distress and being late to sessions. What approach does the PMHNP need to employ with the child to continue making therapeutic progress?

  A. Remaining quiet until the child is ready to talk
  B. Using communication techniques that are expressive
  C. Inviting the child’s parents to speak on behalf of the child
  D. All of the above

1 points  

Question 75

1. A 24-year-old female patient presents for her initial appointment with the PMHNP. Which action will the PMHNP take to establish therapeutic alliance? Psychiatric Nursing

  A. Validating the patient’s affect
  B. Asking the patient questions about her main concerns
  C. Establishing a therapy process
  D. All of the above

Alterations in drug metabolism among Asians

Alterations in drug metabolism among Asians

Question

1. Question :

The drug recommended as primary prevention of osteoporosis in men over seventy years is:

Alendronate (Fosamax)

Ibandronate (Boniva)

Calcium carbonate

Raloxifene (Evista)

Question 2. Question :

Alterations in drug metabolism among Asians may lead to:

Slower metabolism of antidepressants, requiring lower doses

Faster metabolism of neuroleptics, requiring higher doses

Altered metabolism of omeprazole, requiring higher doses

Slower metabolism of alcohol, requiring higher doses

Question 3. Question :

Some research supports that testosterone replacement therapy may be indicated in which of the following diagnoses in men?

Age-related decrease in cognitive functioning

Metabolic syndrome

Decreased muscle mass in aging men

All of the above

Question 4. Question :

The chemicals that promote the spread of pain locally include _________.

serotonin

norepinephrine

enkephalin

neurokinin A

Question 5. Question :

The DEA:

Registers manufacturers and prescribes controlled substances

Regulates NP prescribing at the state level

Sanctions providers who prescribe drugs off-label

Provides prescribers with a number they can use for insurance billing

Question 6. Question :

The trial period to determine effective anti-inflammatory activity when starting a patient on aspirin for RA is _____.

forty-eight hours Alterations in drug metabolism among Asians

four to six days

four weeks

two months

Question 7. Question :

The route of excretion of a volatile drug will likely be:

The kidneys

The lungs

The bile and feces

The skin

Question 8. Question :

Compelling indications for an ACE inhibitor as treatment for hypertension based on clinical trials include:

ORDER A FREE-PLAGIARISM PAPER HERE

Pregnancy

Renal parenchymal disease

Stable angina

Dyslipidemia

Question 9. Question :

The American Diabetic Association has recommended which of the following tests for ongoing management of diabetes?

Fasting blood glucose

Hemoglobin A1c

Thyroid function tests

Electrocardiograms

Question 10. Question :

The angiotensin converting enzyme (ACE) inhibitor lisinopril is a known teratogen. Teratogens cause Type ____ ADR.

A

B

C

D

Question 11. Question :

The goals of therapy when prescribing HRT include reducing:

Cardiovascular risk

Risk of stroke or other thromboembolic event

Breast cancer risk

Vasomotor symptoms

Question 12. Question :

Patients who have angina, regardless of class, who are also diabetic should be on:

Nitrates

Beta blockers

ACE inhibitors

Calcium channel blockers

Question 13. Question :

The New York Heart Association and the Canadian Cardiovascular Society have described grading criteria for levels of angina. Angina that occurs with unusually strenuous activity or on walking or climbing stair after meals is:

Class I

Class II

Class III

Class IV

Question 14. Question :

Patients with allergic rhinitis may benefit from a prescription of:

Fluticasone (Flonase)

Cetirizine (Zyrtec)

OTC cromolyn nasal spray (Nasalcrom)

Any of the above

Question 15. Question :

Kyle has Crohn’s disease and has a documented folate deficiency. Drug therapy for folate deficiency anemia is:

Oral folic acid 1 to 2 mg/day

Oral folic acid 1 gm/day

IM folate weekly for at least six months

Oral folic acid 400 mcg daily

Question 16. Question :

The treatment for vitamin B12 deficiency is:

1,000 mcg daily of oral cobalamin

2 gm/day of oral cobalamin

100 mcg/day vitamin B12 IM

500 mcg/dose nasal cyanocobalamin two sprays once a week

Question 17. Question :

Nonadherence is especially common in drugs that treat asymptomatic conditions, such as hypertension. One way to reduce the likelihood of nonadherence to these drugs is to prescribe a drug that:

Has a short half-life so that missing one dose has limited effect

Requires several dosage titrations so that missed doses can be replaced with lower doses to keep costs down

Has a tolerability profile with less of the adverse effects that are considered “irritating,” such as nausea and dizziness

Must be taken no more than twice a day

Question 18. Question :

Type II diabetes is a complex disorder involving:

Absence of insulin production by the beta cells

A suboptimal response of insulin-sensitive tissues in the liver

Increased levels of GLP in the postprandial period

Too much fat uptake in the intestine

Question 19. Question :

Metformin is a primary choice of drug to treat hyperglycemia in type II diabetes because it:

Substitutes for insulin usually secreted by the pancreas

Decreases glycogenolysis by the liver

Increases the release of insulin from beta cells

Decreases peripheral glucose utilization

Question 20. Question :

Gender differences between men and women in pharmacokinetics include:

More rapid gastric emptying so that drugs absorbed in the stomach have less exposure to absorption sites

Higher proportion of body fat so that lipophilic drugs have relatively greater volumes of distribution

Increased levels of bile acids so that drugs metabolized in the intestine have higher concentrations

Slower organ blood flow rates so that drugs tend to take longer to be excreted

Question 21. Question :

If not chosen as the first drug in hypertension treatment, which drug class should be added as the second step because it will enhance the effects of most other agents?

ACE inhibitors

Beta blockers

Calcium channel blockers

Diuretics

Question 22. Question :

A nineteen-year-old male was started on risperidone. Monitoring for risperidone includes observing for common side effects, including:

Bradykinesia, akathisia, and agitation

Excessive weight gain

Hypertension

Potentially fatal agranulocytosis

Question 23. Question :

Levetiracetam has known drug interactions with:

Oral contraceptives

Carbamazepine

Warfarin

Few, if any, drugs

Question 24. Question :

When the total daily insulin dose is split and given twice daily, which of the following rules may be followed?

Give two-thirds of the total dose in the morning and one-third in the evening.

Give 0.3 units/kg of premixed 70/30 insulin, with one-third in the morning and two-thirds in the evening.

Give 50% of an insulin glargine dose in the morning and 50% in the evening.

Give long-acting insulin in the morning and short-acting insulin at bedtime.

Question 25. Question :

Which of the following factors may adversely affect a patient’s adherence to a therapeutic drug regimen?

Complexity of the drug regimen

Patient’s perception of the potential adverse effects of the drugs

Both A and B

Neither A nor B

Question 26. Question :

The time required for the amount of drug in the body to decrease by 50% is called:

Steady state

Half-life

Phase II metabolism

Reduced bioavailability time

Question 27. Question :

Drugs that are absolutely contraindicated in lactating women include:

Selective serotonin reuptake inhibitors

Antiepileptic drugs such as carbamazepine

Antineoplastic drugs such as methotrexate

All of the above

Question 28. Question :

Tobie presents to clinic with moderate acne. He has been using OTC benzoyl peroxide at home with minimal improvement. A topical antibiotic (clindamycin) and a topical retinoid adapalene (Differin) are prescribed. Education of Tobie would include which one of the following instructions?

He should see an improvement in his acne within the first two weeks of treatment.

If there is no response in a week, he should double the daily application of adapalene (Differin).

He may see an initial worsening of his acne that will improve in six to eight weeks.

Adapalene may cause bleaching of clothing.

Question 29. Question :

Martin is a sixty-year-old with hypertension. The first-line decongestant to be prescribed would be:

Oral pseudoephedrine

Oral phenylephrine

Nasal oxymetazoline

Nasal azelastine

Question 30. Question :

A woman who is pregnant and has hyperthyroidism is best managed by a specialty team that will most likely treat her with:

Methimazole.

Propylthiouracil.

Radioactive iodine.

Nothing; treatment is best delayed until after her pregnancy ends.

Question 31. Question :

A twenty-four-year-old male received multiple fractures in a motor vehicle accident that required significant amounts of opioid medication to treat his pain. He is at risk for Type __ ADR when he no longer requires the opioids.

A

C

E

G

Question 32. Question :

Second-generation antihistamines such as loratadine (Claritin) are prescribed for seasonal allergies because they:

Are more effective than first-generation antihistamines

Are less sedating than first-generation antihistamines

Are prescription products and, therefore, are covered by insurance

Can be taken with CNS sedatives, such as alcohol

Question 33. Question :

Steady state is:

The point on the drug concentration curve when absorption exceeds excretion

When the amount of drug in the body remains constant

When the amount of drug in the body stays below the minimum toxic concentration (MTC)

All of the above

Question 34. Question :

Jayla is a nine-year-old who has been diagnosed with migraines for almost two years. She is missing up to a week of school each month. Her headache diary confirms she averages four or five migraines per month. Which of the following would be appropriate?

Prescribe amitriptyline (Elavil) daily, start at a low dose and increase the dose slowly every two weeks until effective in eliminating migraines.

Encourage her mother to give her Excedrin Migraine (aspirin, acetaminophen, and caffeine) at the first sign of a headache to abort the headache.

Prescribe propranolol (Inderal) to be taken daily for at least three months.

Explain that it is rare for a nine-year-old to get migraines and that she needs an MRI to rule out a brain tumor.

Question 35. Question :

Josie is a five-year-old who presents to the clinic with a forty-eight-hour history of nausea, vomiting, and some diarrhea. She is unable to keep fluids down, and her weight is 4 pounds less than her last recorded weight. Besides intravenous (IV) fluids, her exam warrants the use of an antinausea medication. Which of the following would be the appropriate drug to order for Josie?

Prochlorperazine (Compazine)

Meclizine (Antivert)

Promethazine (Phenergan)

Ondansetron (Zofran)

Question 36. Question :

Long-term use of PPIs may lead to:

Hip fractures in at-risk persons

Vitamin B6 deficiency

Liver cancer

All of the above

Question 37. Question :

Many patients self-medicate with antacids. Which patients should be counseled to not take calcium carbonate antacids without discussing with their providers or a pharmacist first?

Patients with kidney stones

Pregnant patients

Patients with heartburn

IN Postmenopausal women

0 of 2.5

Question 38. Question :

Beta blockers treat hypertension because they:

Reduce peripheral resistance.

Vasoconstrict coronary arteries.

Reduce norepinephrine.

Reduce angiotensin II production.

Question 39. Question :

Precautions that should be taken when prescribing controlled substances include:

Faxing the prescription for a Schedule II drug directly to the pharmacy

Using tamper-proof papers for all prescriptions written for controlled drugs

Keeping any presigned prescription pads in a locked drawer in the clinic Alterations in drug metabolism among Asians

Using only numbers to indicate the amount of drug to be prescribed

Question 40. Question :

The tricyclic antidepressants should be prescribed cautiously in patients with:

Eczema

Asthma

Diabetes

Heart disease

Question 41. Question :

An ACE inhibitor and what other class of drug may reduce proteinuria in patients with diabetes better than either drug alone?

Beta blockers

Diuretics

Nondihydropyridine calcium channel blockers

Angiotensin II receptor blockers

Question 42. Question :

Metoclopramide improves GERD symptoms by:

Reducing acid secretion

Increasing gastric pH

Increasing lower esophageal tone

Decreasing lower esophageal tone

Question 43. Question :

Patient education regarding prescribed medication includes:

Instructions written at the high school reading level

Discussion of expected ADRs

How to store leftover medication such as antibiotics

Verbal instructions always in English

Question 44. Question :

If a patient with H. pylori positive PUD fails first-line therapy, the second-line treatment is:

A PPI BID plus metronidazole plus tetracycline plus bismuth subsalicylate for fourteen days

Testing H. pylori for resistance to common treatment regimens

A PPI plus clarithromycin plus amoxicillin for fourteen days

A PPI and levofloxacin for fourteen days

Question 45. Question :

A patient with a COPD exacerbation may require:

Doubling of inhaled corticosteroid dose

Systemic corticosteroid burst

Continuous inhaled beta 2 agonists

Leukotriene therapy

Question 46. Question :

Angela is a black woman who has heard that women of African descent do not need to worry about osteoporosis. What education would you provide Angela about her risk?

She is ; black women do not have much risk of developing osteoporosis due to their dark skin.

Black women are at risk of developing osteoporosis due to their lower calcium intake as a group.

If she doesn’t drink alcohol, her risk of developing osteoporosis is low.

If she has not lost more than 10% of her weight lately, her risk is low.

Question 47. Question :

The role of the nurse practitioner in the use of herbal medication is to:

Maintain competence in the prescribing of common herbal remedies.

Recommend common OTC herbs to patients.

Educate patients and guide them to appropriate sources of care.

Encourage patients to not use herbal therapy due to the documented dangers.

Question 48. Question :

Ray has been diagnosed with hypertension, and an ACE inhibitor is determined to be needed. Prior to prescribing this drug, the nurse practitioner should assess for:

Hypokalemia

Impotence

Decreased renal function

Inability to concentrate

Question 49. Question :

Long-acting beta-agonists received a black box warning from the US Food and Drug Administration due to the:

Risk of life-threatening dermatological reactions

Increased incidence of cardiac events when long-acting beta-agonists are used

Increased risk of asthma-related deaths when long-acting beta-agonists are used

Risk for life-threatening alterations in electrolytes

Question 50. Question :

Off-Label prescribing is:

Regulated by the FDA

Illegal by NPs in all states (provinces)

Legal if there is scientific evidence for the use

Regulated by the DEA

Question 51. Question :

Adam has type I diabetes and plays tennis for his university. He exhibits knowledge deficit about his insulin and his diagnosis. He should be taught that:

He should increase his increase his carbohydrate intake during times of exercise intake during times of exercise.

Each brand of insulin is equal in bioavailability, so buy the least expensive.

Alcohol produces hypoglycemia and can help control his diabetes when taken in small amounts.

If he does not want to learn to give himself injections, he may substitute an oral hypoglycemic to control his diabetes.

Question 52. Question :

Prior to starting antidepressants, patients should have laboratory testing to rule out:

IN Hypothyroidism

Anemia

Diabetes mellitus

Low estrogen levels

0 of 2.5

Question 53. Question :

What impact does developmental variation in renal function has on prescribing for infants and children?

Lower doses of renally excreted drugs may be prescribed to infants younger than six months

Higher doses of water-soluble drugs may need to be prescribed due to increased renal excretion

Renal excretion rates have no impact on prescribing

Parents need to be instructed on whether drugs are renally excreted or not

Question 54. Question :

All diabetic patients with hyperlipidemia should be treated with:

3-hydroxy-3-methylglutaryl coenzyme A (HMG-CoA) reductase inhibitors

Fibric acid derivatives

Nicotinic acid

Colestipol

Question 55. Question :

Treatment failure in patients with PUD associated with H. pylori may be due to:

Antimicrobial resistance

Ineffective antacid

Overuse of PPIs

All of the above

Question 56. Question :

To improve positive outcomes when prescribing for the elderly, the NP should:

Assess cognitive functioning in the elder

Encourage the patient to take a weekly “drug holiday” to keep drug costs down

Encourage the patient to cut drugs in half with a knife to lower costs

All of the above options are

Question 57. Question :

Erik presents with a golden-crusted lesion at the site of an insect bite consistent with impetigo. His parents have limited finances and request the least expensive treatment. Which medication would be the best choice for treatment?

Mupirocin (Bactroban)

Bacitracin and polymixin B (generic double antibiotic ointment)

Retapamulin (Altabax)

Oral cephalexin (Keflex)

Question 58. Question :

Nonselective beta blockers and alcohol create serious drug interactions with insulin because they:

Increase blood glucose levels.

Produce unexplained diaphoresis.

Interfere with the ability of the body to metabolize glucose.

Mask the signs and symptoms of altered glucose levels.

Question 59. Question :

Infants and young children are at higher risk of ADRs due to:

Immature renal function in school-age children

Lack of safety and efficacy studies in the pediatric population

Children’s skin being thicker than adults, requiring higher dosages of topical medication

Infant boys having a higher proportion of muscle mass, leading to a higher volume of distribution

Question 60. Question :

Warfarin resistance may be seen in patients with VCORC1 mutation, leading to:

Toxic levels of warfarin building up

Decreased response to warfarin

Increased risk for significant drug interactions with warfarin

Less risk of drug interactions with warfarin

Question 61. Question :

Jose is a twelve-year-old overweight child with a total cholesterol level of 180 mg/dL and LDL of 125 mg/dL. Along with diet education and recommending increased physical activity, a treatment plan for Jose would include ____ with a reevaluation in six months.

statins

niacin

sterols

bile acid-binding resins

Question 62. Question :

A potentially life-threatening adverse response to ACE inhibitors is angioedema. Which of the following statements is true about this adverse response?

Swelling of the tongue and hoarseness are the most common symptoms.

It appears to be related to a decrease in aldosterone production.

The presence of a dry, hacky cough indicates a high risk for this adverse response.

Because it takes time to build up a blood level, it occurs after being on the drug for about one week.

Question 63. Question :

Amber is a twenty-four-year-old who has had migraines for ten years. She reports a migraine on average of once a month. The migraines are effectively aborted with naratriptan (Amerge). When refilling Amber’s naratriptan education would include which of the following?

Naratriptan will interact with antidepressants, including selective serotonin reuptake inhibitors and St John’s Wort, and she should inform any providers she sees that she has migraines.

Continue to monitor her headaches; if the migraine is consistently happening around her menses, then there is preventive therapy available.

Pregnancy is contraindicated when taking a triptan.

All the given options are correct.

Question 64. Question :

One of the main drug classes used to treat acute pain is NSAIDs. They are used due to which of the following reasons? Alterations in drug metabolism among Asians

They have less risk for liver damage than acetaminophen.

Inflammation is a common cause of acute pain.

They have minimal GI irritation.

Regulation of blood flow to the kidney is not affected by these drugs.

Question 65. Question :

A patient has been prescribed silver sulfadiazine (Silvadene) cream to treat burns on his or her leg. Normal adverse effects of silver sulfadiazine cream include:

Transient leukopenia on days two to four that should resolve

Worsening of burn symptoms briefly before resolution

A red, scaly rash that will resolve with continued use

Hypercalcemia

Question 66. Question :

Sulfonylureas may be added to a treatment regimen for type II diabetics when lifestyle modifications and metformin are insufficient to achieve target glucose levels. Sulfonylureas have been moved to Step 2 therapy because they:

Increase endogenous insulin secretion.

Have a significant risk for hypoglycemia.

Address the insulin resistance found in type II diabetics.

Improve insulin binding to receptors.

Question 67. Question :

Narcotics are exogenous opiates. They act by ______.

inhibiting pain transmission in the spinal cord

attaching to receptors in the afferent neuron to inhibit the release of substance P

blocking neurotransmitters in the midbrain

increasing beta-lipoprotein excretion from the pituitary

Question 68. Question :

Patients who are on or who will be starting chronic corticosteroid therapy need monitoring of __________.

serum glucose

stool culture

folate levels

vitamin B12

Question 69. Question :

Henry presents to clinic with a significantly swollen, painful great toe and is diagnosed with gout. Of the following, which would be the best treatment for Henry?

High-dose colchicines

Low-dose colchicines

High-dose aspirin

Acetaminophen with codeine

Question 70. Question :

Jaycee has been on escitalopram (Lexapro) for a year and is willing to try tapering off of the selective-serotonin reuptake inhibitors. What is the initial dosage adjustment when starting a taper off antidepressants?

Change the dose to every other day dosing for a week.

Reduce the dose by 50% for three to four days.

Reduce the dose by 50% every other day.

Escitalopram (Lexapro) can be stopped abruptly due to its long half-life.

Question 71. Question :

Christy has exercise and mild persistent asthma and is prescribed two puffs of albuterol fifteen minutes before exercise and as needed for wheezing. One puff per day of beclomethasone (Qvar) is also prescribed. Teaching regarding her inhalers includes which one of the following?

She should use one to two puffs of albuterol per day to prevent an attack, with no more than eight puffs per day.

Beclomethasone needs to be used every day to treat her asthma.

She should report any systemic side effects she is experiencing, such as weight gain.

She should use the albuterol MDI immediately after her corticosteroid MDI to facilitate bronchodilation.

Question 72. Question :

Asthma exacerbations at home are managed by the patient by:

Increasing the frequency of beta 2 agonists and contacting his or her provider

Doubling inhaled corticosteroid dose

Increasing the frequency of beta 2 agonists

Starting montelukast (Singulair)

Question 73. Question :

A woman with an intact uterus should not be prescribed:

Estrogen/progesterone combination

Intramuscular (IM) medroxyprogesterone (Depo Provera)

Estrogen alone

Androgens

Question 74. Question :

The drug recommended as primary prevention of osteoporosis in women over seventy years old is:

Alendronate (Fosamax)

Ibandronate (Boniva)

Calcium carbonate

Raloxifene (Evista)

Question 75. Question :

Patient education when prescribing the vitamin D3 derivative calcipotriene for psoriasis includes:

Applying calcipotriene thickly to affected psoriatic areas two to three times a day

Applying a maximum of 100 grams of calcipotriene per week

Not using calcipotriene in combination with its topical corticosteroids

Augmenting calcipotriene with the use of coal-tar products

Question 76. Question :

Both ACE inhibitors and some angiotensin-II receptor blockers have been approved in treating:

Hypertension in diabetic patients

Diabetic nephropathy

Both A and B

Neither A nor B

Question 77. Question :

Scott is presenting for follow-up on his lipid panel. He had elevated total cholesterol, elevated triglycerides, and an LDL of 122 mg/dL. He has already implemented diet changes and increased physical activity. He has mildly elevated liver studies. An appropriate next step for therapy would be:

Atorvastatin (Lipitor)

Niacin (Niaspan)

Simvastatin and ezetimibe (Vytorin)

Gemfibrozil (Lopid)

Question 78. Question :

Hypoglycemia can result from the action of either insulin or an oral hypoglycemic. Signs and symptoms of hypoglycemia include:

“Fruity” breath odor and rapid respiration

Diarrhea, abdominal pain, weight loss, and hypertension

Dizziness, confusion, diaphoresis, and tachycardia

Easy bruising, palpitations, cardiac dysrhythmias, and coma

Question 79. Question :

Incorporating IT into a patient encounter takes skill and tact. During the encounter, the provider can make the patient more comfortable with the IT the provider is using by:

Turning the screen around so the patient can see material being recorded

Not placing the computer screen between the provider and the patient

Both A and B

Neither A nor B

Question 80. Question :

Which one of the below-given instructions can be followed for applying a topical antibiotic or antiviral ointment?

Apply thickly to the infected area, spreading the medication well past the borders of the infection

If the rash worsens, apply a thicker layer of medication to settle down the infection

Wash hands before and after application of topical antimicrobials

None of the above

Question 81. Question :

Sadie is a ninety-year-old patient who requires a new prescription. What changes in drug distribution with aging would influence prescribing for Sadie?

Increased volume of distribution

Decreased lipid solubility

Decreased plasma proteins

Increased muscle-to-fat ratio

Question 82. Question :

First-line therapy for treating topical fungal infections such as tinea corporis (ringworm) or tinea pedis (athlete’s foot) would be:

OTC topical azole (clotrimazole, miconazole)

Oral terbinafine

Oral griseofulvin microsize

Nystatin cream or ointment

Question 83. Question :

Which of the following is true about procainamide and its dosing schedule?

It produces bradycardia and should be used cautiously in patients with cardiac conditions that a slower heart rate might worsen.

GI adverse effects are common, so the drug should be taken with food.

Adherence can be improved by using a sustained-release formulation that can be given once daily.

Doses of this drug should be taken evenly spaced around the clock to keep an even blood level.

Question 84. Question :

A nurse practitioner would prescribe the liquid form of ibuprofen for a six-year-old because:

Drugs given in liquid form are less irritating to the stomach.

A six-year-old may have problems swallowing a pill.

Liquid forms of medication eliminate the concern for first-pass effect.

Liquid ibuprofen does not have to be dosed as often as tablet form.

Question 85. Question :

Kristine would like to start HRT to treat the significant vasomotor symptoms she is experiencing during menopause. Education for a woman considering hormone replacement would include:

Explaining that HRT is totally safe if used for a short term

Telling her to ignore media hype regarding HRT

Discussing the advantages and risks of HRT

Encouraging the patient to use phytoestrogens with the HRT

Question 86. Question :

Xi, a fifty-four-year-old female, has a history of migraine that does not respond well to OTC migraine medication. She is asking to try Maxalt (rizatriptan) because it works well for her friend. Which of the following actions would you take for appropriate decision making?

Prescribe Maxalt, but to monitor the use, only give her four tablets with no refills.

Prescribe Maxalt and arrange to have her observed in the clinic or urgent care with the first dose.

Explain that rizatriptan is not used for postmenopausal migraines and recommend Fiorinal (aspirin and butalbital).

Prescribe sumatriptan (Imitrex) with the explanation that it is the most effective triptan.

Question 87. Question :

The elderly are at high risk of ADRs due to:

Having greater muscle mass than younger adults, leading to higher volume of distribution

The extensive studies that have been conducted on drug safety in this age group

The blood-brain barrier being less permeable, requiring higher doses to achieve therapeutic effect

Age-related decrease in renal function Alterations in drug metabolism among Asians

Question 88. Question :

Jim presents with fungal infection of two of his toenails (onychomycosis). Treatment for fungal infections of the nail includes:

Miconazole cream

Ketoconazole cream

Oral griseofulvin

Mupirocin cream

Question 89. Question :

GLP-1 agonists:

Directly bind to a receptor in the pancreatic beta cell.

Have been approved for monotherapy.

Speed gastric emptying to decrease appetite.

Can be given orally once daily.

Question 90. Question :

Prophylactic use of bisphosphonates is recommended for patients with early osteopenia related to long-term use of which of the following drugs?

Selective estrogen-receptor modulators

Aspirin

Glucocorticoids

Calcium supplements

Question 91. Question :

Inadequate vitamin D intake can contribute to the development of osteoporosis by:

Increasing calcitonin production

Increasing calcium absorption from the intestine

Altering calcium metabolism

Stimulating bone formation

Question 92. Question :

Which of the following statements is true about age and pain?

Use of drugs that depend heavily on the renal system for excretion may require dosage adjustments in very young children.

Among the NSAIDs, indomethacin is the preferred drug because of lower adverse effects profiles than other NSAIDs.

Older adults who have dementia probably do not experience much pain due to loss of pain receptors in the brain.

Acetaminophen is especially useful in both children and adults because it has no effect on platelets and has fewer adverse effects than NSAIDs.

Question 93. Question :

Selective estrogen receptor modifiers (SERMs) treat osteoporosis by selectively:

Inhibiting magnesium resorption in the kidneys

Increasing calcium absorption from the gastrointestinal (GI) tract

Acting on the bone to inhibit osteoblast activity

Selectively acting on the estrogen receptors in the bone

Question 94. Question :

When a patient is on selective-serotonin reuptake inhibitors:

The complete blood count must be monitored every three to four months

Therapeutic blood levels must be monitored every six months after a steady state is achieved.

Blood glucose must be monitored every three to four months.

There is no laboratory monitoring required.

Question 95. Question :

Patients whose total dose of prednisone exceed 1 gram will most likely need a second prescription for _________.

metformin, a biguanide to prevent diabetes

omeprazole, a proton pump inhibitor to prevent peptic ulcer disease

naproxen, an NSAID to treat joint pain

furosemide, a diuretic to treat fluid retention

Question 96. Question :

The ongoing monitoring of patients over the age sixty-five years taking alendronate (Fosamax) or any other bisphosphonate is:

Annual dual energy X-ray absorptiometry (DEXA) scans

Annual vitamin D level

Annual renal function evaluation

Electrolytes every three months

Question 97. Question :

ORDER HERE

Anticholinergic agents, such as benztropine (Cogentin), may be given with a phenothiazine to:

Reduce the chance of tardive dyskinesia.

Potentiate the effects of the drug.

Reduce the tolerance that tends to occur.

Increase CNS depression.

Question 98. Question :

Diagnostic criteria for diabetes include:

Fasting blood glucose greater than 140 mg/dl on two occasions

Postprandial blood glucose greater than 140 mg/dl

Fasting blood glucose 100 to 125 mg/dl on two occasions

Symptoms of diabetes plus a casual blood glucose greater than 200 mg/dl

Question 99. Question :

Insulin preparations are divided into categories based on onset, duration, and intensity of action following subcutaneous inject. Which of the following insulin preparations has the shortest onset and duration of action?

Insulin lispro

Insulin glulisine

Insulin glargine

Insulin detemir

Question 100. Cynthia is taking valproate (Depakote) for seizures and would like to get pregnant. What advice would you give her?

Valproate is safe during all trimesters of pregnancy.

She can get pregnant while taking valproate, but she should take adequate folic acid.

Valproate is not safe at any time during pregnancy.

Valproate is a known teratogen but may be taken after the first trimester if necessary.

Primary method of continuous quality improvement (CQI) in health care, and what is its underlying premise

Due at 12PM CST, Monday, November 6, 2017

HCA 340 500

Discussion Board Assignment #10

Chapters12—Research:  How Health Care Advances and 13–Future of Health Care

Answer the following questions on Research: How Health Care Advances:

  1.            Definitions of the quality of medical care are no longer left to clinicians who decide for themselves what technical performance constitutes “good care.”  Name one (1) other dimension of quality care and why is it important? 
  2.            What is the primary method of continuous quality improvement (CQI) in health care, and what is its underlying premise?
  3.            Since an ever-increasing amount of research is funded by commercial companies, what is your opinion on a requirement for authors of scientific studies who receive income from these companies to disclose funding sources in their scientific publications?

    ORDER A FREE-PLAGIARISM PAPER HERE

Future of Health Care Questions:

  1.            Today’s not-for-profit hospitals and health systems are commonly multi-billion dollar tax-exempt organizations. What is your position on the new federal scrutiny upon, and reporting requirements for justifying the charitable missions of not-for-profit hospitals and health systems?
  2.            In 1999, the Institute of medicine report, To Err is Human,generated a brief flurry of concerns about avoidable hospital deaths.  While progress has been made in addressing system errors and deficiencies, it remains inconsistent across the nation’s hospitals.  Fourteen years after this report, is it time for the professions, payers and the public to demand corrections of system problems in an accountable, transparent and publicly disclosed manner?  What form can or should these demands take?

A descriptive and reflective discussion of how the new tool or intervention may be integrated into practice that is supported by sound research

Diabetes topic 1

Identify a research or evidence-based article that focuses comprehensively on a specific intervention or new diagnostic tool for the treatment of diabetes in adults or children.

In a paper of 750-1,000 words, summarize the main idea of the research findings for a specific patient population. Research must include clinical findings that are current, thorough, and relevant to diabetes and the nursing practice.

Prepare this assignment according to the APA guidelines found in the APA Style Guide, located in the Student Success Center. An abstract is not required.

ORDER A FREE-PLAGIARISM PAPER HERE

This assignment uses a grading rubric. Instructors will be using the rubric to grade the assignment; therefore, students should review the rubric prior to beginning the assignment to become familiar with the assignment criteria and expectations for successful completion of the assignment.

You are required to submit this assignment to Turnitin. Refer to the directions in the Student Success Center. Only Word documents can be submitted to Turnitin.

Part 2 powerpoint

Based on the summary of research findings identified from the Evidence-Based Project—Paper on Diabetes that describes a new diagnostic tool or intervention for the treatment of diabetes in adults or children, complete the following components of this assignment:

Develop a PowerPoint presentation (a title slide, 6-12 slides, and a reference slide; no larger than 2 MB) that includes the following:

A brief summary of the research conducted in the Evidence-Based Project – Paper on Diabetes.

A descriptive and reflective discussion of how the new tool or intervention may be integrated into practice that is supported by sound research.

While APA format is not required for the body of this assignment, solid academic writing is expected, and in-text citations and references should be presented using APA documentation guidelines, which can be found in the APA Style Guide, located in the Student Success Center.

You are not required to submit this assignment to Turnitin, unless otherwise directed by your instructor. If so directed, refer to the Student Success Center for directions. Only Word documents can be submitted to Turnitin.